A 2020 InsightsIAS Textbookwise Test 12 PDF

You might also like

Download as pdf or txt
Download as pdf or txt
You are on page 1of 81

Total Marks : 200

Online Prelims TEST - 12 (TEXTBOOK)


( InsightsIAS Mock Test Series for UPSC Preliminary Exam 2020 ) Mark Scored : 0

1 Consider the following statements about the initiative Half-earth by E.O. Wilson.
1. It is a call to protect half of the earth by designating it a human-free natural reserve to preserve
biodiversity.
2. The Half-Earth Project Map helps locate where place-based species conservation activities are
needed the most to save the bulk of Earth’s species.

Select the correct answer using the codes below.


A. 1 only
B. 2 only
C. Both 1 and 2
D. None of the above

Your Answer :
Correct Answer : C

Answer Justification :

Justification: Statement 1: Half-Earth is a call to protect half the land and sea in order to manage
sufficient habitat to safeguard the bulk of biodiversity.

Advances in technology now allow us to comprehensively map the geospatial location and
distribution of the species of our planet at high enough resolution to drive decision-making about
where we have the best opportunity to protect the most species. This is the work of the Half-Earth
Project.

This is based on Half-Earth: Our Planet's Fight for Life, a 2016 book by the biologist E. O. Wilson.

Wilson noted that the term "Half-Earth" was coined for this concept by Tony Hiss in his Smithsonian
article "Can the World Really Set Aside Half the Planet for Wildlife?"

The Half-Earth Project Map is a high-resolution, dynamic world map and decision-support tool that
guides where place-based species conservation activities are needed the most to save the bulk of
Earth’s species, including humans.

Q Source: https://www.half-earthproject.org/discover-half-earth/

https://www.insightsonindia.com/2019/09/30/insights-into-editorial-why-the-latest-ipcc-report-on-oce
ans-matters/

2 Consider the following statements.


1. The Government of India has made 2030 as its target to eliminate tuberculosis (TB) from the
country, based on the global target set by the UN.
2. India ranks first in the world as far as TB-related mortality is concerned.
3. TB is the leading cause of morbidity and mortality among People Living with HIV and those with
HIV are more likely to contract TB than non-HIVs.

Select the correct answer using the codes below.


A. 1 and 2 only
Total Marks : 200
Online Prelims TEST - 12 (TEXTBOOK)
( InsightsIAS Mock Test Series for UPSC Preliminary Exam 2020 ) Mark Scored : 0

B. 3 only
C. 1 and 3 only
D. 1, 2 and 3

Your Answer :
Correct Answer : B

Answer Justification :

Justification: Statement 1: United Nations has marked 2030 as a global target to eliminate TB
worldwide but the Government of India has made 2025 as its target to eliminate tuberculosis from
the country.

Context: ‘TB Harega Desh Jeetega Campaign’ has been launched with an aim to eliminate
tuberculosis from India.

Campaign aims to improve and expand the reach of TB care services across the country, by
2022.
Three pillars of the campaign- clinical approach, public health component and active
community participation.
Why is Tuberculosis a major cause of concern?
TB is one of the leading causes of death worldwide and the leading cause from a single
infectious agent, ranking above HIV/AIDS.
TB is an infectious disease caused by the bacillus Mycobacterium tuberculosis.
It typically affects the lungs (pulmonary TB) but can also affect other sites.
The disease is spread when people who are sick with pulmonary TB expel bacteria into the
air, for example by coughing.

Statement 2 and 3: India TB report 2019- key findings:

1. The number of HIV-infected people who go on to develop Tuberculosis (TB) is increasing in


India.

2. TB is the leading cause of morbidity and mortality among People Living with HIV (PLHIV).
This group is 21 times more likely to develop TB than persons without the virus.

3. India is the third-highest HIV burden country in the world, with an adult prevalence of 0.22
per cent.

4. India ranks second in the world as far as TB-related mortality is concerned.

5. The highest percentage of patients who tested positive for TB and were also infected with HIV
came from Nagaland (15.6 per cent), followed by Karnataka (10 per cent).
Total Marks : 200
Online Prelims TEST - 12 (TEXTBOOK)
( InsightsIAS Mock Test Series for UPSC Preliminary Exam 2020 ) Mark Scored : 0

6. The co-morbidity of TB don’t come in form of HIV only. Diabetes and tobacco-related ailments
too play a role.

7. Overall, the TB burden in India is highest in Uttar Pradesh.

8. Percentage of pediatric tuberculosis (TB among the population aged less than 15 years) cases
have also slightly gone up.

Q Source:
https://www.insightsonindia.com/2019/09/26/insights-daily-current-affairs-pib-26-september-2019/

3 The sculptural tradition in Gandhara had the confluence of

A. Bactria, Parthia and the local Gandhara tradition


B. Persian, Sri Lankan and Kashmiri tradition
C. Pahari, Karla and Kashmiri tradition
D. Bactria, Karla and Sri Lankan traditions

Your Answer :
Correct Answer : A

Answer Justification :

Learning: The first century CE onwards, Gandhara (now in Pakistan), Mathura in northern India
and Vengi in Andhra Pradesh emerged as important centres of art production. Buddha in the
symbolic form got a human form in Mathura and Gandhara.

Developed between the 1st century BCE and the 7th century CE, the Gandhara style, of
Greco-Roman origin, seems to have flourished largely during the Kushan dynasty and was
contemporaneous with an important but dissimilar school of Kushan art at Mathura (Uttar
Pradesh).
The local sculptural tradition at Mathura became so strong that the tradition spread to other
parts of northern India. The best example in this regard is the stupa sculptures found at
Sanghol in the Punjab.
The Buddha image at Mathura is modelled on the lines of earlier Yaksha images whereas in
Gandhara it has Hellenistic features.

Q Source: Based on UPSC Mains 2019 + Page 30: 11th NCERT: Introduction to Arts

4 Which of the following bodies in its Trade and Development Report 2019 has recommended for the
adoption of a unitary taxation system for the Multi-National Enterprises (MNEs)?

A. World Economic Forum


B. United Nations Conference on Trade and Development (UNCTAD)
C. World Trade Organization (WTO)
Total Marks : 200
Online Prelims TEST - 12 (TEXTBOOK)
( InsightsIAS Mock Test Series for UPSC Preliminary Exam 2020 ) Mark Scored : 0

D. World Bank (WB)

Your Answer :
Correct Answer : B

Answer Justification :

Justification: The proposal:

The profits of MNEs are generated collectively at the group level. Hence, unitary taxation
should be applied by combining it with a global minimum effective corporate tax rate on all
MNE profits.

Such an approach would simplify the global taxation system and is expected to increase tax
revenues for all countries.

Need for and significance:

There was a dire need for this change, as the current international corporate taxation norms
consider affiliates of MNEs as independent entities and treat taxable transactions between
different entities of MNEs as unrelated.

The fiscal revenues of a country could be augmented through fair taxation of the digital
economy.

Concerns:

The tax-motivated illicit financial flows of MNEs are estimated to deprive developing countries
of $50 billion to $200 billion a year in terms of the fiscal revenues.

Background:

The international tax system needs a paradigm shift. The rules devised over 80 years ago treat the
different parts of a multinational enterprise as if they were independent entities, although they also
give national tax authorities powers to adjust the accounts of these entities. This creates a perverse
incentive for multinationals to create ever more complex groups in order to minimise taxes,
exploiting the various definitions of the residence of legal persons and the source of income. While
states may attempt to combat these strategies, they also compete to offer tax incentives, many of
which facilitate such techniques to undermine other countries’ taxes.

Q Source:
https://www.insightsonindia.com/2019/09/28/insights-daily-current-affairs-pib-28-september-2019/

5 The largest human source of carbon dioxide emissions is from


Total Marks : 200
Online Prelims TEST - 12 (TEXTBOOK)
( InsightsIAS Mock Test Series for UPSC Preliminary Exam 2020 ) Mark Scored : 0

A. Industrial manufacturing
B. Combustion of fossil fuels
C. Agricultural fields
D. Clearing of forests

Your Answer :
Correct Answer : B

Answer Justification :

Learning: Nearly 87 percent of all human-produced carbon dioxide emissions come from the
burning of fossil fuels like coal, natural gas and oil.

The remainder results from the clearing of forests and other land use changes (9%), as well as some
industrial processes such as cement manufacturing (4%)

Coal is the most carbon intensive fossil fuel. For every tonne of coal burned, approximately 2.5
tonnes of CO2e are produced

Q Source: Frequently in news

6 India does NOT hold joint military exercises with which of the following nations?
1. China
2. Kazakhstan
3. Sri Lanka

Select the correct answer using the codes below.


A. 1 only
B. 2 and 3 only
C. 1 and 2 only
D. None of the options given above are correct

Your Answer :
Correct Answer : D

Answer Justification :

Justification: India holds military exercises with all the above. Some of the nations with whom
India does not hold these in South Asia are Pakistan and Bhutan.

Some of the prominent nations with which these are held are USA, China, Russia, Japan, SL, UK,
France, Singapore etc.

Here is a useful compilation.

Indian Navy Joint Exercises

Varuna naval exercise : France and India.


Total Marks : 200
Online Prelims TEST - 12 (TEXTBOOK)
( InsightsIAS Mock Test Series for UPSC Preliminary Exam 2020 ) Mark Scored : 0

SLINEX-Sri Lanka India


INDRA - India and Russia
Exercise Malabar - United States and India.
Simbex - Indian Navy with Republic of Singapore Navy
IBSAMAR with the Brazil and South African navies.
KONKAN - A bilateral Naval Exercise between Indian Navy and Royal Navy of Britain

Indian Army Joint Exercises

Mithra Shakti exercise- India and Sri Lanka.


Hand-in-Hand -India-China joint military training exercise
Exercise Shakti - India and France armies
Exercise Nomadic Elephant- Indian Army exercises with the Mongolian Army.
Exercise Yuddh Abhyas is a series of joint exercises between the Indian and United States
Armies since 2005.
Surya Kiran - India Nepal joint military exercise
LAMITYE- India and Seychelles
Prabal Dostyk - India Kazhakhstan Joint exercise

Check out the full list here https://en.wikipedia.org/wiki/List_of_exercises_of_the_Indian_Army

Q Source: Frequently in news

7 Article 263 of the Constitution of India provides for the establishment of an Inter-State Council. What
are the functions of the council?
1. It is a permanent secretariat for cooperation between the various planning organs of State
Governments.
2. It considers recommendations for the better coordination of policy and matters of general interest
to the states.
3. It decides on the allocation of Finance Commission approved financial outlay for each state.

Select the correct answer using the codes below.


A. 1 only
B. 1 and 2 only
C. 2 only
D. 2 and 3 only

Your Answer :
Correct Answer : C

Answer Justification :

Justification: Statement 1: The inter-state council is not a permanent constitutional body for
coordination between the states and union government. It can be established ‘at any time’ if it
appears to the President that the public interests would be served by the establishment of such a
council.

Statement 2 and 3: The Council is a recommendatory body to investigate and discuss subjects, in
Total Marks : 200
Online Prelims TEST - 12 (TEXTBOOK)
( InsightsIAS Mock Test Series for UPSC Preliminary Exam 2020 ) Mark Scored : 0

which some or all of the states or the union government have a common interest.

Learning: The Council shall consist of:

Prime minister who is the chairman.


Chief ministers of all states who are members.
Chief ministers of union territories and administrators of UTs as members.
Six union ministers of cabinet rank in the union council of ministers nominated by the prime
minister are also members.

Q Source: 11th NCERT: Indian Constitution at Work

8 With reference to Project NETRA, consider the following statements.


1. The project was initiated by ISRO.
2. Under NETRA, a string of satellites will be placed in the geo-stationary orbits to support NAVIC
satellites.
3. The project will give India its own capability in space situational awareness (SSA) in detecting and
averting space debris.

Select the correct answer using the codes below.


A. 1 and 3 only
B. 2 and 3 only
C. 1 and 2 only
D. 1, 2 and 3

Your Answer :
Correct Answer : A

Answer Justification :

Justification: ISRO has initiated ‘Project NETRA’ – an early warning system in space to detect
debris and other hazards to Indian satellites.

What is Project NETRA (Network for space object Tracking and Analysis)?

Under the project, the ISRO plans to put up many observational facilities: connected radars,
telescopes; data processing units and a control centre.

They can, among others, spot, track and catalogue objects as small as 10 cm, up to a range of
3,400 km and equal to a space orbit of around 2,000 km.

Significance of the project:

1. The project will give India its own capability in space situational awareness (SSA) like the
other space powers — which is used to ‘predict’ threats from debris to Indian satellites.
Total Marks : 200
Online Prelims TEST - 12 (TEXTBOOK)
( InsightsIAS Mock Test Series for UPSC Preliminary Exam 2020 ) Mark Scored : 0

2. NETRA’s eventual goal is to capture the GEO, or geostationary orbit, scene at 36,000 km
where communication satellites operate.

3. The effort would make India a part of international efforts towards tracking, warning about
and mitigating space debris.

Q Source:
https://www.insightsonindia.com/2019/09/27/insights-daily-current-affairs-pib-27-september-2019/

9 Consider the following statements.


1. Formation of ozone consumes oxygen.
2. Nitrogen-fixing by most bacteria happens in presence of oxygen.
3. Oxygen is consumed when the plant combines the carbon dioxide and the water by using the Sun's
energy.

Select the correct answer using the codes below.


A. 1 only
B. 1 and 2 only
C. 2 and 3 only
D. 1, 2 and 3

Your Answer :
Correct Answer : A

Answer Justification :

Justification: Statement 1: Free oxygen atoms can recombine to form oxygen molecules but if a
free oxygen atom collides with an oxygen molecule, it joins up, forming ozone. Ozone molecules can
also be decomposed by ultraviolet radiation into a free atom and an oxygen molecule.

Statement 2: It is largely an anaerobic process. Enzymes responsible for nitrogenase action are very
susceptible to destruction by oxygen. For this reason, many bacteria cease production of the
enzyme in the presence of oxygen.

So, many nitrogen-fixing organisms exist only in anaerobic conditions, respiring to draw down
oxygen levels, or binding the oxygen with a protein such as leghemoglobin.

Statement 3: This is the process of photosynthesis that produces oxygen instead of consuming it.

During photosynthesis green plants manufacture the sugar molecules fructose and glucose. Green
plants use energy from sunlight to build sugar molecules from carbon dioxide and water.

Q Source: General science covered in 9th Geography NCERT

10 With reference to the ‘Special Report on the Ocean and Cryosphere in a Changing Climate’ recently
published by IPCC, consider the following statements.
Total Marks : 200
Online Prelims TEST - 12 (TEXTBOOK)
( InsightsIAS Mock Test Series for UPSC Preliminary Exam 2020 ) Mark Scored : 0

1. Since 1990s, the rate of ocean warming has more than doubled.
2. A new phenomena of Marine heatwaves has surfaced in the 21st CE.

Select the correct answer using the codes below.


A. 1 only
B. 2 only
C. Both 1 and 2
D. None of the above

Your Answer :
Correct Answer : A

Answer Justification :

Justification: The report was prepared following an IPCC Panel decision in 2016 to prepare three
Special Reports and follows the Special Reports on Global Warming of 1.5°C, and on Climate
Change and Land (SRCCL).

Key findings:

1. Over the 21st century, the ocean is projected to transition to unprecedented conditions with
increased temperatures, further ocean acidification, marine heatwaves and more frequent
extreme El Niño and La Niña events.

2. The global ocean has warmed unabated since 1970 and has taken up more than 90% of the
excess heat in the climate system.

3. Since 1993, the rate of ocean warming has more than doubled.

4. Marine heatwaves have very likely doubled in frequency since 1982 and are increasing in
intensity.

5. Floods will become more frequent and severe in the mountainous and downstream areas of
the Indus, Ganges and Brahmaputra river basins, because of an increase in extreme
precipitation events.

Q Source:
https://www.insightsonindia.com/2019/09/27/insights-daily-current-affairs-pib-27-september-2019/

11 As per the Koeppen classification, which of the following regions of India has a Steppe Climate?

A. Central India
B. Parts of North-western India and leeward side of Western Ghats
Total Marks : 200
Online Prelims TEST - 12 (TEXTBOOK)
( InsightsIAS Mock Test Series for UPSC Preliminary Exam 2020 ) Mark Scored : 0

C. North-eastern India
D. Coastal regions of south-eastern India

Your Answer :
Correct Answer : B

Answer Justification :

Learning: Steppe climate will have lower and irregular rainfall. North-eastern India receives either
abundant rainfall or gets too cold for being called a steppe climate.

Q Source: Chapter 4: 11th NCERT: India Physical Geography

12 Consider the following statements.


1. Paikas were the traditional land-owning militia of Odisha and served as warriors
2. Paikas rebelled against the British when the British administration curtailed their rights to access
Minor Forest Produce and impose a tax on forest entry.
Total Marks : 200
Online Prelims TEST - 12 (TEXTBOOK)
( InsightsIAS Mock Test Series for UPSC Preliminary Exam 2020 ) Mark Scored : 0

Select the correct answer using the codes below.


A. 1 only
B. 2 only
C. Both 1 and 2
D. None of the above

Your Answer :
Correct Answer : A

Answer Justification :

Justification: Paikas had been recruited since the 16th century by kings in Odisha from a variety of
social groups to render martial services in return for rent-free land (nish-kar jagirs) and
titles. They were the traditional land-owning militia of Odisha and served as warriors.

How the rebellion began?

1. When armies of the East India Company overran most of Odisha in 1803, the Raja of Khurda
lost his primacy and the power and prestige of the Paikas went on a decline.

2. The British were not comfortable with these aggressive, warlike new subjects and set up a
commission under Walter Ewer to look into the issue.

3. The commission recommended that the hereditary rent-free lands granted to the Paikas be
taken over by the British administration and this recommendation was zealously adhered to.
They revolted against the British.

4. Bakshi Jagabandhu Bidyadhar Mohapatra Bharamarbar Rai, the highest-ranking


military general of King of Khorda Mukund Dev II, led the Paikas to join the uprising.

5. However, the rebellion had several other underlying causes – like the rise in the price of salt,
abolition of the cowrie currency for payment of taxes and an overtly extortionist land revenue
policy.

6. Although initially the Company struggled to respond they managed to put down the rebellion
by May 1817. Many of the Paik leaders were hung or deported. Jagabandhu surrendered in
1825.

Q Source:
https://www.insightsonindia.com/2019/09/26/insights-daily-current-affairs-pib-26-september-2019/

13 Consider the following statements.


Under the Constitution of the United States of America (USA)
Total Marks : 200
Online Prelims TEST - 12 (TEXTBOOK)
( InsightsIAS Mock Test Series for UPSC Preliminary Exam 2020 ) Mark Scored : 0

1. The President can be removed from office for treason, bribery, or other high crimes and
misdemeanors.
2. The House of Representatives (Lower House) has the sole power of impeachment while the Senate
(Upper House) has the sole power to try all impeachments.
3. The Chief Justice of the US Supreme Court has the duty of overseeing impeachment trials in the
Senate.

Select the correct answer using the codes below.


A. 1 and 2 only
B. 2 and 3 only
C. 1, 2 and 3
D. 1 and 3 only

Your Answer :
Correct Answer : C

Answer Justification :

Justification: Grounds for impeachment:

1. The President can be removed from office for “treason, bribery, or other high crimes and
misdemeanors”.

2. Essentially, it means an abuse of power by a high-level public official. This does not
necessarily have to be a violation of an ordinary criminal statute. Historically, in the US, it
has encompassed corruption and other abuses, including trying to obstruct judicial
proceedings.

The process:

1. It begins with an investigation by a House committee. If they find that there is enough
evidence of wrongdoing, it will refer the matter to the full House.

2. HOUSE VOTE: When the full House votes, if one or more of the articles of impeachment gets
a majority vote, the President is impeached. Next, the proceedings move to the Senate.

3. SENATE TRIAL & VOTE: The Senate holds a trial, overseen by the chief justice of the
Supreme Court. A team of lawmakers from the House, known as managers, play the role of
prosecutors. The President has defence lawyers, and the Senate serves as the jury. If at least
two-thirds of the Senators present find the President guilty, he is removed and the Vice
President takes over as President.

Q Source:
Total Marks : 200
Online Prelims TEST - 12 (TEXTBOOK)
( InsightsIAS Mock Test Series for UPSC Preliminary Exam 2020 ) Mark Scored : 0

https://www.insightsonindia.com/2019/09/26/insights-daily-current-affairs-pib-26-september-2019/

14 Writing in the journal Young India in 1931, who said, "I cannot possibly bear the idea that a man who
has got wealth should get the vote but a man who has got character but no wealth or literacy should
have no vote….for the crime of being a poor man…"?

A. Jawahar Lal Nehru


B. M.K. Gandhi
C. Rajendra Prasad
D. Maulana Abul Kalam Azad

Your Answer :
Correct Answer : B

Answer Justification :

Learning: Young India, Harijan and Hind Swaraj were some of the most famous works of Gandhi.

Young India was a weekshed - a weekly paper or journal.

He used Young India to spread his unique ideology and thoughts regarding the use of nonviolence
in organising movements and to urge readers to consider, organise, and plan for India's eventual
independence from Britain.

The journal was reprinted in USA by the India Home Rule League of America.

Q Source: AR: Page 33: NCERT Class VI Social and Political Life - I

15 Equator passes through which of these African countries?


1. Congo
2. Kenya
3. Niger
4. Namibia

Select the correct answer using the codes below.


A. 1 and 2 only
B. 2 and 3 only
C. 1 and 4 only
D. 3 and 4 only

Your Answer :
Correct Answer : A

Answer Justification :

Justification: Africa is the only continent through which all the major latitudinal lines pass - Tropic
of Cancer, Tropic of Capricorn and Equator.
Total Marks : 200
Online Prelims TEST - 12 (TEXTBOOK)
( InsightsIAS Mock Test Series for UPSC Preliminary Exam 2020 ) Mark Scored : 0

Equator runs through Gabon, Republic of Congo, Democratic Republic of Congo, Uganda, Kenya
and Somalia.

Tropic of Cancer passes through Mali, Algeria, Niger, Libya, Egypt and Mauritania.

The Tropic of Capricorn passes through the countries of Namibia, Botswana, South Africa and
Mozambique.

Q Source: Map-based questions: Africa

16 A Bengali writer, her book titled “Amar Jiban” is the first known autobiography written by an Indian
woman. She is?

A. Ismat Chugtai
B. Begum Rukhaiya Hussain
C. Deepa Bandopadhyay
D. Rashsundari Devi

Your Answer :
Correct Answer : D

Answer Justification :

Learning: At the age of 60, Rashsundari Devi wrote her autobiography in Bangla in 1860. She and
a few others like (Rukhaiya Hussain) were some the most prominent figures in the literary history of
women in colonial India.

Option A: Ismat Chugtai (post-colonial period) was a liberal Muslim writer and considered to be a
path-breaking woman writer of the region.

Option B: Begum Rukhaiya Hussain wrote Sultana’s Dream in 1905 to practise her English skills
when she was merely 25 years old. This story imagined a woman called Sultana who reaches a place
called Ladyland. Ladyland is a place where women took similar roles as men.

Q Source: Page 60: NCERT 7th: Social and Economic Life – II

17 With reference to the International Tribunal for the Law of the Sea (ITLOS), consider the following:
1. It is an organ of the United Nations adjudicating disputes arising out of the interpretation and
Total Marks : 200
Online Prelims TEST - 12 (TEXTBOOK)
( InsightsIAS Mock Test Series for UPSC Preliminary Exam 2020 ) Mark Scored : 0

application of the UN Convention on the Law of the Seas (UNCLOS).


2. Each state party of the ITLOS nominates a representative to become a member of ITLOS.

Which of the above is/are correct?


A. 1 only
B. 2 only
C. Both 1 and 2
D. None

Your Answer :
Correct Answer : D

Answer Justification :

Justification: Statement 1: The Tribunal is an independent judicial body but maintains close links
with the United Nations.

The Tribunal and the UN have entered into agreement concerning cooperation and relationship.

UN has granted the Tribunal observer status in the General Assembly.

Statement 2: The Tribunal is composed of 21 independent members, elected from among persons
enjoying the highest reputation for fairness and integrity and of recognized competence in the field
of the law of the sea.

Recently, law expert Neeru Chadha has become the first Indian women to elected to the
International Tribunal for the Law of the Seas (ITLOS).
Neeru has also earned a distinction of becoming the second Indian woman to occupy a top
position at the United Nations after Lakshmi Pandit who served as the President of the United
Nations General Assembly.

Q Source: International Organizations

18 Though the President of India is not a member of either House of Parliament and does not sit in the
Parliament to attend its meetings, she is an integral part of the Parliament. This is because
1. A bill passed by both the Houses of Parliament cannot become law without the President’s assent.
2. She performs important functions relating to the proceedings of the Parliament such as summoning
and proroguing both the Houses and even dissolving the Lok Sabha.

Which of the above is/are correct?


A. 1 only
B. 2 only
C. Both 1 and 2
D. None

Your Answer :
Correct Answer : C
Total Marks : 200
Online Prelims TEST - 12 (TEXTBOOK)
( InsightsIAS Mock Test Series for UPSC Preliminary Exam 2020 ) Mark Scored : 0

Answer Justification :

Justification: Statement 1: This makes her an important limb of the law making process and thus
an integral part of Parliament.

Statement 2: She performs certain functions relating to the proceedings of the Parliament, for
example, he summons and prorogues both the Houses, dissolves the Lok Sabha, addresses both the
Houses, issues ordinances when they are not in session, and so on. These are very important
functions and the responsibility to execute them can scarcely be designated to a lower functionary.

Source: Polity

19 Consider the following statements.


National Academic Depository (NAD)
1. is an E-network of libraries in India
2. Facilitates recognition of degrees awarded by one institute by the other
3. Holds the repository of all academic talent in India for better resource utilization

Select the correct answer using the codes below.


A. 1 only
B. 2 and 3 only
C. 2 only
D. 3 only

Your Answer :
Correct Answer : C

Answer Justification :

Justification: NAD is a National System set-up by MHRD and UGC to facilitate Academic
Institutions to Digitally, Securely and Quickly issue Online Academic Awards to the Students
directly in their online NAD Account.

The student can access certificate at any time and authorise employers, banks to view and verify the
certificates.

No more need to carry originals, submit attested photocopies, wait for verification of
documents all the time.
NAD aspires to make the vision of Digital Academic Certificates for every Indian a reality.
NAD promises to do away with difficulties / inefficiencies of collecting, maintaining, and
presenting physical paper certificates.

Q Source: Based on past year UPSC papers

20 Which of these geographical features is most likely to have the lowest mean elevation?

A. Plateaus
B. Islands
Total Marks : 200
Online Prelims TEST - 12 (TEXTBOOK)
( InsightsIAS Mock Test Series for UPSC Preliminary Exam 2020 ) Mark Scored : 0

C. Deltas
D. Plains

Your Answer :
Correct Answer : C

Answer Justification :

Justification: Option A: They are elevated land tops, and maintain a rugged high profile of terrain.

Option B: Islands although situated at sea level can acquire higher elevation due to presence of
features like mountains, volcanoes etc.

Option C: Deltas occur near the sea mouth, i.e. very close to Mean sea level, which can be the
lowest elevation possible.

Option D: Rivers flow further from plains to join the sea. So, plains are usually at a higher elevation
than delta.

Q Source: Page 39 (Figure 6.1): 6th NCERT Geography

21 Consider the following statements about Malaria and its incidence in India.
1. The majority of malaria reporting districts are in India’s eastern and central parts.
2. The Government has set malaria elimination deadline as 2027.
3. Malaria is most commonly transmitted by a male mosquito or female sand-flies.

Select the correct answer using the codes below.


A. 1 and 2 only
B. 3 only
C. 1 and 3 only
D. 2 only

Your Answer :
Correct Answer : A

Answer Justification :

Justification: The World malaria report 2018 draws on data from 87 countries and areas with
ongoing malaria transmission.

Data from 2015–2017 highlight that no significant progress in reducing global malaria cases was
made in this period. There were an estimated 219 million cases and 435 000 related deaths in 2017.

According to the World Malaria Report 2016 (which still remains relevant), India accounts for 89%
of the incidence of malaria in the South-East Asia region.

Statement 1: In India, malaria parasites are found more in the forest areas and plains.

Most malaria cases are mainly concentrated in tribal and remote areas of the country.
Total Marks : 200
Online Prelims TEST - 12 (TEXTBOOK)
( InsightsIAS Mock Test Series for UPSC Preliminary Exam 2020 ) Mark Scored : 0

Six states — Odisha (40%), Jharkhand (20%), Chhattisgarh (20%), Meghalaya, Arunachal
Pradesh and Mizoram (5-7%) report most of the malaria cases in India.
These states, along with tribal areas of Madhya Pradesh and Maharashtra account for 90% of
India’s malaria burden.

Statement 2: The Union Health Ministry and Family Welfare for first time has unveiled National
Strategic Plan (NSP) for Malaria Elimination (2017-22). It has set malaria elimination deadline as
2027, three years ahead of the global deadline (under SDGs).

Statement 3: Malaria is a vector borne disease caused by parasitic protozoans belonging to the
Plasmodium type. It is most commonly transmitted by an infected female Anopheles mosquito.

Q Source: National Strategic Plan (NSP) for Malaria Elimination (2017-22)

22 Which of the following functions are performed by the Lok Sabha?


1. Approval of proposals for taxation
2. Appointment of Chairman of Council of States
3. Approving a constitutional amendment bill
4. Approving the Proclamation of emergency
5. Appointment of members of higher judiciary

Select the correct answer using the codes below.


A. 1, 3 and 4 only
B. 1, 2 and 5 only
C. 2, 3, 4 and 5 only
D. 1, 2, 3, 4 and 5

Your Answer :
Correct Answer : A

Answer Justification :

Justification: Statement 1: Such matters come under money bills. Money bills can only be
introduced in LS, not RS.

Statement 2: This is done by Rajya Sabha. If the question had asked the Vice-President, you should
have chosen this statement as correct. Lok Sabha serves as the collegiums for electing the VP.

Statement 3: Both houses can do so. A bill for this purpose, if defeated by any house, lapses and
cannot be presented in a joint sitting of the houses.

Statement 4: Both houses can approve this proclamation. Lok Sabha can also revoke an emergency.

Statement 5: In case of SC, this is done by the President on the advice of a collegiums consisting of
the CJI and some senior-most judges of HC. LS, however, takes part in the impeachment of
judiciary.

Q Source: 11th NCERT: Constitution at work


Total Marks : 200
Online Prelims TEST - 12 (TEXTBOOK)
( InsightsIAS Mock Test Series for UPSC Preliminary Exam 2020 ) Mark Scored : 0

23 With reference to coronal explosions, consider the following:


1. It produces large radiations and particles which may significantly influence the atmosphere of the
star.
2. It has been observed only in the Sun.
3. If the flare from such an explosion reaches earth, it can damage electrical equipment and power
grids.

Select the correct answer using the codes below.


A. 1 and 2 only
B. 1 and 3 only
C. 2 only
D. 1, 2 and 3

Your Answer :
Correct Answer : B

Answer Justification :

Justification & Learning: Coronal explosion is an unusually large release of plasma and magnetic
field from the solar corona.

They often follow solar flares and are normally present during a solar prominence eruption.
The plasma is released into the solar wind.

Statement 2: Recently, India’s first dedicated space observatory AstroSat along with other space
and earth-based observatories have detected a massive coronal explosion on Proxima Centauri,
sun’s closest star neighbour.

Statement 3: The massive coronal explosion has sent out powerful solar flare having energy about
100 times a typical solar flare.

If such a massive flare occurs in our Sun, it might have a devastating effect on power grids,
interrupt broadcasts and electricity, affect electronic instruments, and cause excess UV radiation in
space.

Statement 1: Such powerful flaring may have produced large radiations and particles which may
significantly influence the atmosphere of the Proxima Centauri b and affect its habitability.

Q Source: https://www.swpc.noaa.gov/phenomena/coronal-mass-ejections

24 Consider the following statements.


Assertion (A): Pallava rulers issued their charters only in Prakrit.
Reason (R): Pallava rulers were against the dominance of Brahmanism and banned the use of
Sanskrit for all official works.

In the context of the above, which of these is correct?


A. A is correct, and R is an appropriate explanation of A.
B. A is correct, but R is not an appropriate explanation of A.
Total Marks : 200
Online Prelims TEST - 12 (TEXTBOOK)
( InsightsIAS Mock Test Series for UPSC Preliminary Exam 2020 ) Mark Scored : 0

C. A is correct, but R is incorrect.


D. Both A and R are incorrect.

Your Answer :
Correct Answer : D

Answer Justification :

Justification: The early Pallava rulers from 250 A.D. to 350 A.D. issued their charters in Prakrit.
Important among them were Sivaskandavarman and Vijayaskandavarman.

The second line of Pallava rulers who ruled between 350 A.D. and 550 A.D. issued their charters in
Sanskrit. So, R is wrong.

One instance is mentioned below:

The Hirahadagali copper plate (Bellary District) record in Prakrit is dated in the 8th year of
Sivaskanda Varman to 283 CE and confirms the gift made by his father who is described
merely as "Bappa-deva" (revered father) or Boppa. It will thus be clear that this dynasty of the
Prakrit charters beginning with "Bappa-deva" were the historical founders of the Pallava
dominion in South India.

The Hirahadagalli Plates were found in Hirehadagali, Bellary district and is one of the earliest
copper plates in Karnataka and belongs to the reign of early Pallava ruler Shivaskanda Varma.
Pallava King Sivaskandavarman of Kanchi of the early Pallavas ruled from 275 to 300 CE, and
issued the charter in 283 CE in the 8th year of his reign.

Q Source: Revision: History 12th NCERT

25 Telecom Regulatory Authority of India (TRAI) is empowered to


1. Regulate telecom market to ensure fair competition
2. Fix tariffs for telecom services in India

Which of the above is/are correct?


A. 1 only
B. 2 only
C. Both 1 and 2
D. None

Your Answer :
Correct Answer : C

Answer Justification :

Justification: Statement 1: The TRAI is an independent regulator of the telecommunications


business in India. Its main purpose is to deliver a fair and transparent environment for fair
Total Marks : 200
Online Prelims TEST - 12 (TEXTBOOK)
( InsightsIAS Mock Test Series for UPSC Preliminary Exam 2020 ) Mark Scored : 0

competition in telecom market.

It came into existence by the Act of the Parliament in 1997. It was established in wake of entry of
private sector in telecom industries after Government had constituted the National Telecom Policy
(NTP) to attract domestic and FDI investment in the telecommunication sector.

Statement 2: TRAI also fixes or revises the tariffs for telecom services in India.

TRAI also has punitive mandate. Recently it had recommended Department of Telecom (DoT) to
impose combined penalty of 3,050 crore rupees on Bharti Airtel, Vodafone and Idea cellular.

Q Source: Frequently in news

26 Land revenue was a major economic mainstay of the Mughal Empire. Consider the following with
reference to it.
1. Both cultivated and cultivable lands were measured in each province.
2. Office of the diwan was responsible for supervising the fiscal system of the empire.
3. Terms such as ‘Jama’ and ‘Hasil’ were often used to measure the rank of revenue collecting officers.

Select the correct answer using the codes below.


A. 1 and 2 only
B. 3 only
C. 1 and 3 only
D. 1, 2 and 3

Your Answer :
Correct Answer : A

Answer Justification :

Justification: Statement 1: It was vital for the state to create an administrative system to ensure
control over agricultural production, and to fix and collect revenue from across the empire.

Efforts to measure lands continued under subsequent emperors. Aurangzeb instructed his revenue
officials to prepare annual records of the number of cultivators in each village. Yet not all areas
were measured successfully.

Statement 2: This system included the office of the diwan who was responsible for supervising the
fiscal system of the empire. Thus revenue officials and record keepers penetrated the agricultural
domain and became a decisive agent in shaping agrarian relations.

Akbar ordered revenue collectors that they should strive to make cultivators pay in cash, the option
of payment in kind was also to be kept open. While fixing revenue, the attempt of the state was to
maximise its claims.

Statement 3: The land revenue arrangements consisted of two stages – first, assessment and then
actual collection. The jama was the amount assessed and hasil, the amount collected.

Q Source: 12th NCERT: Themes in India History


Total Marks : 200
Online Prelims TEST - 12 (TEXTBOOK)
( InsightsIAS Mock Test Series for UPSC Preliminary Exam 2020 ) Mark Scored : 0

27 What is a fast neutron reactor?

A. It is a type of fusion reactor where nuclear fuel is replenished from the source continuously.
B. It is a fission reactor that does not need neutron moderators.
C. It is a rector where fission reactions are used to energize fusion reactions.
D. It is a mobile reactor that can be connected to grids in remote areas.

Your Answer :
Correct Answer : B

Answer Justification :

Learning: It is a type in which the nuclear fission chain reaction is sustained by fast neutrons. It is
also known as a fast reactor.

Such a reactor needs no neutron moderator like normal water, which serves such a purpose in
thermal reactors.

With these types of reactors it is possible to solve the major ecological problem of reprocessing and
deactivation of accumulated radioactive waste, at same time provide much needed energy.

It will lead to the solution of the five key problems safety, shortage of fuel, competitiveness,
reprocessing and re-fabricating used nuclear fuel and radioactive waste and enforcing non-
proliferation of fission materials and weapons technologies.

Q Source: Technologies that are making the mark

28 On the south-western coasts of India, you are most likely to come across which of these forests?

A. Tropical deciduous forests


B. Mangrove forests
C. Tropical rainforests
D. Montane forests

Your Answer :
Correct Answer : C

Answer Justification :

Learning: These forests are restricted to heavy rainfall areas of the Western Ghats (near south-
western coasts) and the island groups of Lakshadweep, Andaman and Nicobar, upper parts of
Assam and Tamil Nadu coast.

They are at their best in areas having more than 200 cm of rainfall with a short dry season. The
trees reach great heights.

Since the region is warm and wet throughout the year, it has a luxuriant vegetation of all kinds –
trees, shrubs, and creepers giving it a multilayered structure.
Total Marks : 200
Online Prelims TEST - 12 (TEXTBOOK)
( InsightsIAS Mock Test Series for UPSC Preliminary Exam 2020 ) Mark Scored : 0

There is no definite time for trees to shed their leaves. As such, these forests appear green all the
year round.

Q Source: NCERT 9th Geography

29 What is referred to as the “Informalization of the Indian workforce”?


Total Marks : 200
Online Prelims TEST - 12 (TEXTBOOK)
( InsightsIAS Mock Test Series for UPSC Preliminary Exam 2020 ) Mark Scored : 0

A. A decline in the number of self-employment or entrepreneurship opportunities


B. Job loss in traditional sectors such as farming
C. No mutual recognition of certification of the Indian workers across industries
D. None of the above

Your Answer :
Correct Answer : D

Answer Justification :

Learning: Informalisation of workforce refers to the situation wherein there is a continuous decline
in the percentage of workforce in the formal sector and a simultaneous increase in informal
contract or casual jobs.

For e.g construction sector jobs that are not full time and irregular and do not carry any social
security with them.

It is also called as casualization of workforce.

Informalization in India seems to have been rooted in rigid labour laws; failure of private sector to
generate gainful employment; automation of work etc.

Q Source: Indian Economic Development: 11th NCERT

30 Consider the following statements.


1. Alluvial soils are suitable for the growth of sugarcane and pulse crops.
2. Alluvial soils are poor in potash, lime and phosphoric acid.

Which of the above is/are correct?


A. 1 only
B. 2 only
C. Both 1 and 2
D. None of the above

Your Answer :
Correct Answer : A

Answer Justification :

Justification: Alluvial soils as a whole are very fertile. Mostly these soils contain adequate
proportion of potash, phosphoric acid and lime which are ideal for the growth of sugarcane, paddy,
wheat and other cereal and pulse crops.

Due to its high fertility, regions of alluvial soils are intensively cultivated and densely populated.

Soils in the drier areas are more alkaline and can be productive after proper treatment and
irrigation.
Total Marks : 200
Online Prelims TEST - 12 (TEXTBOOK)
( InsightsIAS Mock Test Series for UPSC Preliminary Exam 2020 ) Mark Scored : 0

Q Source: India Physical Geography: 11th NCERT

31 Consider the following about “Trade Secrets”.


1. It is an informal term for ‘patents’ when recognized only within national borders.
2. It confers a competitive commercial advantage to those who possess such information.
3. It is regulated under TRIPS agreement.

Select the correct answer using the codes below.


A. 1 only
B. 2 and 3 only
C. 2 only
D. 1, 2 and 3

Your Answer :
Correct Answer : B

Answer Justification :

Justification: Statement 1 and 2: Protection of undisclosed information (or trade secrets) is any
information that has been intentionally treated as secret and is capable of commercial application
with an economic interest.

It protects information that confers a competitive advantage to those who possess such information,
provided such information is not readily available with or discernible by the competitors (for e.g.
unique psychological marketing techniques not known to all). It is different from patents, which is a
recognized innovation.

Statement 3: TRIPS Agreement is, by its coverage, the most comprehensive international
instrument on IPRs, dealing with all types of IPRs, with the sole exception of breeders’ rights. IPRs
covered under the TRIPS agreement are: copyrights, trade secrets, patents, GIs, industrial designs
etc.

Q Source: DIPP Website: Current Affairs: Often in news

32 With reference to the “Rationalization Thesis” given by Weber, consider the following statements.
1. It explains the process through which magic, supernatural and religious ideas lose cultural
importance in a society
2. It helps understand the rise of modern corporations and bureaucracy.

Which of the above is/are correct?


A. 1 only
B. 2 only
C. Both 1 and 2
D. None

Your Answer :
Correct Answer : C
Total Marks : 200
Online Prelims TEST - 12 (TEXTBOOK)
( InsightsIAS Mock Test Series for UPSC Preliminary Exam 2020 ) Mark Scored : 0

Answer Justification :

Justification: Rationalization was the process through which magic, supernatural and religious
ideas lose cultural importance in a society and ideas based on science and practical calculation
become dominant.

For example, in modern societies science has rationalized our understanding of weather patterns.
Science explains weather patterns as a result of interaction between physical elements like wind-
speed and direction, air and water temperatures, humidity, etc.

Rationalization also involves the development of forms of social organization devoted to the
achievement of precise goals by efficient means. It is this type of rationalization that we see in the
development of modern business corporations and of bureaucracy. These are organizations
dedicated to the pursuit of defined goals by calculated, systematically administered means.

There are other aspects of the thesis as well, but we will restrict it here for brevity.

Q Source: AR: UPSC Past year papers

33 Why the membership of Nuclear Supplier’s Group (NSG) is important for India?
1. Its membership will increase India’s access to state-of-the-art technology from the other members
of the Group.
2. India will get an opportunity to voice it’s concern if in case of change in the provision of the binding
rules of NSG.

Select the correct answer using the codes below.


A. 1 only
B. 2 only
C. Both 1 and 2
D. None of the above

Your Answer :
Correct Answer : A

Answer Justification :

Justification: At the Bloomberg Global Business Forum in New York, Prime Minister Narendra
Modi made a pitch for India’s membership of the Nuclear Suppliers Group (NSG).

What is NSG?

Brought in 1974– in response to the Indian nuclear test (smiling Buddha).


It is a Multilateral export control regime.
It is a Group of nuclear supplier countries that seek to prevent nuclear proliferation by
controlling the export of materials, equipment and technology that can be used to
manufacture nuclear weapons.
The NSG first met in November 1975 in London, and is thus popularly referred to as
the “London Club”.
Total Marks : 200
Online Prelims TEST - 12 (TEXTBOOK)
( InsightsIAS Mock Test Series for UPSC Preliminary Exam 2020 ) Mark Scored : 0

Statement 2: It is NOT a formal organization, and its guidelines are not binding.
Decisions, including on membership, are made by consensus.

Why the membership is important for India?

1. Membership will increase India’s access to state-of-the-art technology from the other
members of the Group.

2. Access to technology and being allowed to produce nuclear equipment will give a boost to
the Make in India program. That will, in turn, boost the economic growth of our country.

3. As per India’s INDC under the Paris Climate agreement, we have committed to reducing
dependence on fossil fuels and ensuring that 40% of its energy is sourced from renewable and
clean sources. In order to achieve this target, we need to scale up nuclear power production.
This can only happen if India gains access to the NSG.

4. India will get an opportunity to voice it’s concern if in case of change in the provision of the
NSG guidelines.

Other Benefits associated with NSG membership- Once admitted, an NSG member state gets:

1. Timely information on nuclear matters.

2. Contributes by way of information.

3. Has confirmed credentials.

4. Can act as an instrument of harmonization and coordination.

5. Is part of a very transparent process.

Learning: Criteria for membership:

1. Ability to supply items (including items in transit) covered by the annexes to Parts 1 and 2 of
the NSG Guidelines;

2. Adherence to the Guidelines and action in accordance with them;


Total Marks : 200
Online Prelims TEST - 12 (TEXTBOOK)
( InsightsIAS Mock Test Series for UPSC Preliminary Exam 2020 ) Mark Scored : 0

3. Enforcement of a legally based domestic export control system which gives effect to the
commitment to act in accordance with the Guidelines;

4. Full compliance with the obligations of one or more of nuclear non-proliferation agreement.

5. Support of international efforts towards non-proliferation of weapons of mass destruction and


of their delivery vehicle.

Q Source:
https://www.insightsonindia.com/2019/09/26/insights-daily-current-affairs-pib-26-september-2019/

34 What is often referred to as “Market failures” by economists is actually

A. Lack of equilibrium between supply and demand in the market


B. Failure of markets to allocate resources efficiently
C. Unequal distribution of resource use between private and social goods
D. Too much competition at the market place leading to failure of individual firms

Your Answer :
Correct Answer : B

Answer Justification :

Learning: The ideology of Laissez fairez is based on the invisible hand of free markets. Such
markets which work on self-interest of individuals help in the efficient allocation of resources in
perfectly competitive markets.

But, such markets seldom exist due to:

Monopolies and other institutional factors


Instable private sector
Economic shocks
High poverty and lack of purchasing power

Markets fail to deliver optimum results in the above cases and the intervention of the state becomes
a must in these cases, for e.g. delivering public health infrastructure in rural areas where people
can’t afford expensive private care or where such care may not be available.

Q Source: General concepts in economics: Sometimes seen in news

35 Consider the following capitals of North-eastern states of India.


1. Aizwal
2. Imphal
3. Kohima
4. Itanagar
Total Marks : 200
Online Prelims TEST - 12 (TEXTBOOK)
( InsightsIAS Mock Test Series for UPSC Preliminary Exam 2020 ) Mark Scored : 0

What is the correct order of these capitals from south to North?


A. 1234
B. 2314
C. 3214
D. 1342

Your Answer :
Correct Answer : A

Answer Justification :

Q Source: Map-based questions: India


Total Marks : 200
Online Prelims TEST - 12 (TEXTBOOK)
( InsightsIAS Mock Test Series for UPSC Preliminary Exam 2020 ) Mark Scored : 0

36 Based on the provisions of the National Water Policy, what is the water allocation priority, in
decreasing order?

A. Drinking water, Irrigation, Hydropower, Navigation, Industrial and other uses


B. Drinking water, Hydropower, Navigation, Irrigation, Industrial and other uses
C. Irrigation, Industrial and other uses, Drinking water, Hydropower and Navigation
D. Drinking water, Irrigation, Navigation, Industrial and other uses, Hydropower

Your Answer :
Correct Answer : A

Answer Justification :

Justification: Major provisions under the National Water policy are:

1. Envisages to establish a standardized national information system with a network of data


banks and data bases.

2. Guidelines for the safety of storage dams and other water-related structures.

3. Regulate exploitation of groundwater.

4. Setting water allocation priorities in the following order: Drinking


water, Irrigation, Hydropower, Navigation, Industrial and other uses.

5. The water rates for surface water and ground water should be rationalized with due regard to
the interests of small and marginal farmers.

6. The policy also deals with participation of farmers and voluntary agencies, water quality,
water zoning, conservation of water, flood and drought management, erosion etc.

Q Source:
https://www.insightsonindia.com/2019/09/26/insights-daily-current-affairs-pib-26-september-2019/

37 Arrange these major Gulfs from West to East.


1. Gulf of Guinea
2. Gulf of Aden
3. Gulf of Mexico
4. Gulf of Mannar

Select the correct answer using the codes below.


A. 3124
Total Marks : 200
Online Prelims TEST - 12 (TEXTBOOK)
( InsightsIAS Mock Test Series for UPSC Preliminary Exam 2020 ) Mark Scored : 0

B. 2143
C. 1342
D. 3214

Your Answer :
Correct Answer : A

Answer Justification :

Justification: Gulf of Aden lies near Horn of Africa on the eastern side. Gulf of Mannar lies further
north-east in India.

Q Source: Map-based questions

38 Privy Purse system allowed

A. Government to confiscate private land based on the principle of ‘eminent domain’


B. Local administration to abolish zamindari from erstwhile permanent settlement regions
C. British government to appropriate a fraction of India’s revenues for certain years after
independence to maintain financial continuity
D. Princely state rulers to retain certain private property and grant in heredity

Your Answer :
Correct Answer : D

Answer Justification :

Learning: Abolition of Privy Purse was a major issue in post-independent India.

The integration of India was preceded by an assurance that after the dissolution of princely rule, the
then rulers’ families would be allowed to retain certain private property, and given a grant in
heredity or government allowance, measured on the basis of the extent, revenue and potential of
Total Marks : 200
Online Prelims TEST - 12 (TEXTBOOK)
( InsightsIAS Mock Test Series for UPSC Preliminary Exam 2020 ) Mark Scored : 0

the merging state.

This grant was called the Privy Purse. At the time of accession, there was little criticism of these
privileges since integration and consolidation was the primary aim.

Yet, hereditary privileges were not consonant with the principles of equality and social and
economic justice laid down in the Constitution of India. Nehru had expressed his
dissatisfaction over the matter time and again.
Following the 1967 elections, Indira Gandhi supported the demand that the government
should abolish privy purses. Morarji Desai, however, called the move morally wrong and
amounting to a ‘breach of faith with the princes’.
The government tried to bring a Constitutional amendment in 1970, but it was not passed in
Rajya Sabha. It then issued an ordinance which was struck down by the Supreme Court.
Indira Gandhi made this into a major election issue in 1971 and got a lot of public support.
Following its massive victory in the 1971 election, the Constitution was amended to remove
legal obstacles for abolition of ‘privy purse’.

Q Source: Important topics from UPSC past year papers

39 The soil productivity of the tropical rain forest is lower when compared to that of the temperate
forests. This is because of

A. Intense leaching of soil in tropical rain forests


B. Dense vegetation Cover in tropical rain forests
C. Practice of slash and burn agriculture in many tropical regions
D. Low microbial activity in tropical regions

Your Answer :
Correct Answer : A

Answer Justification :

Learning: The soil is poor in nutrients and acidic due to frequent leaching by heavy rains.

Frequent rains wash away the top soil leaving only certain mineral and organic remains. So, if these
forests are cleared, it will not yield tremendous vegetation growth as seen in tropical forests for a
long time.

However, rainforests are also notable for replenishing the soil quickly with dead organic matter
(e.g. leafs that fall from trees).

Q Source: 9th NCERT Geography

40 Consider the following about State Finance Commissions (SFCs).


1. They are constitutional bodies.
2. Their mandate is to recommend devolution of tax funds from the Consolidated Fund of State
Government to local bodies.
3. Their recommendations must be approved by the Union Finance Commission, in order to be
Total Marks : 200
Online Prelims TEST - 12 (TEXTBOOK)
( InsightsIAS Mock Test Series for UPSC Preliminary Exam 2020 ) Mark Scored : 0

effective.
4. They are established by the President from time to time.

Select the correct answer using the codes below.


A. 1, 2 and 4 only
B. 2 and 3 only
C. 1 and 2 only
D. 1, 3 and 4 only

Your Answer :
Correct Answer : C

Answer Justification :

Justification: Statement 1 and 2: As per 73rd and 74th CA, under Part XI and XII and Article 243(I)
and 243 (Y) of the Constitution, a State Finance Commission(SFC), is to be appointed after every 5
years, to;

Recommend devolution of tax funds from the Consolidated Fund of State Government to Local
Bodies
Suggest measures for augmenting their Own Resources by determining which Taxes, Duties,
Tolls and Fees which may be assigned to or appropriated by PRIs and ULBs
The Grants-in-Aid to Local Bodies from the consolidated fund of the state
The measures needed to improve the financial position of the Local Bodies
Estimation of Revenue Gap of Local Bodies, separately for PRIs and ULBs,

Statement 3: The Union FC does not play any role with respect to either constitution or working of
the SFCs. It may only help guide the overall functioning of the SFCs by specific recommendations
(that are not binding on the SFCs).

Statement 4: The Governor constitutes them. President constitutes the UFC.

Q Source: 11th NCERT: Indian Constitution at Work

41 Who among the following is often called as “the first lady of Indian cinema”?

A. Shabana Azmi
B. Nargis
C. Meena Kumari
D. Devika Rani

Your Answer :
Correct Answer : D

Answer Justification :

Justification: Devika Rani was one of the first women from Indian cinema to be formally educated
in films
Total Marks : 200
Online Prelims TEST - 12 (TEXTBOOK)
( InsightsIAS Mock Test Series for UPSC Preliminary Exam 2020 ) Mark Scored : 0

Her awards include the Padmashri (1958), Dadasaheb Phalke Award (1970) and the Soviet
Land Nehru Award (1990).

Dadasaheb Phalke Award for 2019 will be presented to Amitabh Bachchan.

It is the country’s highest film honour conferred for “Outstanding contribution for the growth
and development of Indian cinema”.

The Award was introduced by the government in 1969 and it was awarded for the first time
to Devika Rani, “the first lady of Indian cinema”.

Q Source:
https://www.insightsonindia.com/2019/09/25/insights-daily-current-affairs-pib-25-september-2019/

42 Consider the following with reference to the East India Association organized in London.
1. It was founded by Dadabhai Naoroji.
2. The organization aimed to present correct information about India to the British Public and voice
Indian Grievances.
3. The East India Association incorporated the National Indian Association before the Komagata Maru
incident.

Select the correct answer using the codes below.


A. 1 and 2 only
B. 2 and 3 only
C. 3 only
D. 1 and 3 only

Your Answer :
Correct Answer : A

Answer Justification :

Justification: Statement 1: Dadabhai Naoroji initiated establishment of East India Association, at


London. It was one of the predecessor organizations of the Indian National Congress in 1867.

Statement 2: Over the course of its existence, the Association would listen to lectures from a wide
range of Indian and British men and women on matters ranging from the economic development of
India to literature to suffrage.

It superseded the London Indian Society and was a platform for discussing matters and ideas about
India, and to provide representation for Indians to the Government.

Statement 3: The East India Association incorporated the National Indian Association in 1949 and
became the Britain, India and Pakistan Association. In 1966 it amalgamated with the former India
Society, now Royal India, Pakistan and Ceylon Society, to become the Royal Society for India,
Total Marks : 200
Online Prelims TEST - 12 (TEXTBOOK)
( InsightsIAS Mock Test Series for UPSC Preliminary Exam 2020 ) Mark Scored : 0

Pakistan and Ceylon.

Q Source: AR: Past year UPSC papers

43 Which of the following is\are salient features of the Value Added Tax (VAT) system?
1. It is a form of direct taxation.
2. It is based on self-assessment system and provides the facility of tax credit and tax refund.
3. A major shortcoming of the VAT system is that it promotes cascading of taxes.

Select the correct answer using the codes below.


A. 1 only
B. 2 and 3 only
C. 2 only
D. 3 only

Your Answer :
Correct Answer : C

Answer Justification :

Justification: Statement 1: VAT is a form of indirect taxation. VAT is a broad-based tax as it covers
the value added to each commodity by a firm during all stages of production and distribution.

Statement 2: VAT is based on value added principle. Value added can be obtained either by adding
payments to factors of production (i.e. , wages+rent+interest+profit) or deducting cost of inputs
from sales revenue.

VAT is a substitute for sales tax, hotel tax. contract tax and entertainment tax.

VAT is based on self-assessment system and provides the facility of tax credit and tax refund.

The meaning of this will become clear from the explanation below.

Statement 3: Cascading effect of taxes occurs when the government taxes product on those prices
which have already been taxed in some other form.

For e.g. a shirt requires cotton as raw material. Suppose cotton is priced at Rs. 100 and taxed at
10%. Final value of cotton for producers is 110. He uses this to make a shirt, where he adds a value
of Rs. 40, and prices the product at Rs. 150. If the same tax of 10% applies on the shirt again, the
price becomes 165. Note that cotton has been effectively taxed twice. This is called cascading when
the actual tax burden is much that is evident from the tax rate.

So, VAT taxes product only on the value added, i.e. Rs 40, and not the entire Rs. 150, which was the
price of the shirt. Producers self-assess this situation and report the value added (and price of raw
material), based on which the IT department sends them back tax credit.

Q Source: GST is based on VAT model


Total Marks : 200
Online Prelims TEST - 12 (TEXTBOOK)
( InsightsIAS Mock Test Series for UPSC Preliminary Exam 2020 ) Mark Scored : 0

44 Consider the following statements.


1. He was an imperialist and called himself ‘a Bengal tiger’.
2. He came to India with a determination to launch a forward policy in order to make ‘the British
Empire in India’ into ‘the British Empire of India’.

The above refer to?


A. Lord Dufferin
B. Lord Lytton
C. Lord Dupleix
D. Lord Wellesley

Your Answer :
Correct Answer : D

Answer Justification :

Learning: The appointment of Richard Colley Wellesley as Governor General marks an epoch in the
history of British India.

Wellesley came to India with a determination to launch a forward policy in order to make ‘the
British Empire in India’ into ‘the British Empire of India’. The system that he adopted to achieve his
object is known as the ‘Subsidiary Alliance’.

Wellesley’s annexations and the vast military expenditure that he had authorized alarmed the court
of directors of the East India Company. In 1805 he was recalled, and soon afterward he was
threatened with impeachment, although two years later he refused an offer of the foreign secretary-
ship.

Q Source: AR: Past year UPSC papers

45 “Class Conflict” is central to which of the following ideologies?

A. Socialism
B. Marxism
C. Capitalism
D. Fascism

Your Answer :
Correct Answer : B

Answer Justification :

Learning: Class conflict is the tension or antagonism which exists in society due to competing
socioeconomic interests and desires between people of different classes.

The view that the class struggle provides the lever for radical social change for the majority is
central to the work of Karl Marx and the anarchist Mikhail Bakunin.
Total Marks : 200
Online Prelims TEST - 12 (TEXTBOOK)
( InsightsIAS Mock Test Series for UPSC Preliminary Exam 2020 ) Mark Scored : 0

Class conflict can take many different forms: direct violence, such as wars fought for resources and
cheap labor; indirect violence, such as deaths from poverty, starvation etc.

Labour union against capitalists is an important example of class struggle. It is a typical example
class conflict within capitalism. This class conflict is seen to occur primarily between the
bourgeoisie and the proletariat, and takes the form of conflict over hours of work, value of wages,
division of profits, cost of consumer goods, the culture at work, control over parliament or
bureaucracy, and economic inequality.

Q Source: AR: Past year UPSC papers

46 Hyperspectral imaging can be useful in


1. Finding oil fields
2. Monitoring crop health
3. Diagnosis of retinopathy
4. Removing defects and foreign material in processed food

Select the correct answer using the codes below.


A. 1 and 2 only
B. 2, 3 and 4 only
C. 3 and 4 only
D. 1, 2, 3 and 4

Your Answer :
Correct Answer : D

Answer Justification :

Concept: Whereas the human eye sees colour of visible light in mostly three bands (red, green, and
blue), spectral imaging divides the spectrum into many more bands.

This technique of dividing images into bands can be extended beyond the visible.
In hyperspectral imaging, the recorded spectra have fine wavelength resolution and cover a
wide range of wavelengths. This can cover very fine imaging.

Justification: Statement 1: Known as spectral signatures, these 'fingerprints' enable identification


of the materials that make up a scanned object. For example, a spectral signature for oil helps
geologists find new oil fields.

Statement 2: Although the cost of acquiring hyperspectral images is typically high, for specific
crops and in specific climates, hyperspectral remote sensing use is increasing for monitoring the
development and health of crops. This is because the finer colours that indicate crop health can be
mapped by hyperspectral imaging.

Statement 3: Researchers in Montraal are working to test the use of hyperspectral photography in
the diagnosis of retinopathy and macular edema before damage to the eye occurs.

Statement 4: In the food processing industry, hyperspectral imaging, combined with intelligent
Total Marks : 200
Online Prelims TEST - 12 (TEXTBOOK)
( InsightsIAS Mock Test Series for UPSC Preliminary Exam 2020 ) Mark Scored : 0

software, enables digital sorters (also called optical sorters) to identify and remove defects and
foreign material (FM) that are invisible to traditional camera and laser sorters.

Q Source: Revision: Current Affairs (last 2 years):


https://timesofindia.indiatimes.com/india/isro-launches-indias-first-hyperspectral-imaging-sat-along-
with-30-foreign-sats/articleshow/66859630.cms

47 The climate and weather of a region largely depends on its


1. Distance from equator
2. Distance from International Date Line
3. Height above Mean Sea Level (MSL)
4. Proximity with major water bodies

Select the correct answer using the codes below.


A. 1, 3 and 4 only
B. 2 and 4 only
C. 1 and 3 only
D. 1, 2, 3 and 4

Your Answer :
Correct Answer : A

Answer Justification :

Justification: Statement 1: This refers to latitude. Places farther from equator are likely to have
lesser rainfall and a colder climate.

Statement 2: This indirectly refers to the longitude of a place, which has no bearing on a place’s
weather or climate.

Statement 3: Altitude affects the temperature of a place. Places at higher MSL are more likely to be
colder than those at lower MSL.

Statement 4: This refers to continentality. Places close to large lakes or sea are more likely to have
moderate weather due to the blowing of land and sea breezes.

Q Source: 9th Geography NCERT

48 Migratory birds use many natural signals to navigate or find direction. Which of these may be
generally used by migratory birds in navigation?
1. Apparent changing position of Sun
2. Apparent movement of stars with respect to earth
3. Magnetic field of earth

Select the correct answer using the codes below.


A. 1 and 2 only
B. 2 only
Total Marks : 200
Online Prelims TEST - 12 (TEXTBOOK)
( InsightsIAS Mock Test Series for UPSC Preliminary Exam 2020 ) Mark Scored : 0

C. 1 and 3 only
D. 1, 2 and 3

Your Answer :
Correct Answer : D

Answer Justification :

Justification: Some migratory birds travel as much as 15000 km to escape the extreme climatic
conditions at home. Generally they fly high where the wind flow is helpful and the cold conditions
allow them to disperse the heat generated by their flight muscles.

It seems that these birds have a built—in sense of direction and know in which direction to travel.
Some birds probably use landmarks to guide them. Many birds may be guided by the sun during the
day and stars at night. There is some evidence that birds may use the magnetic field of the earth to
find direction. They have enhanced sensory perceptions which enable them to track changes in
magnetic field and move accordingly.

Q Source: 8th Science NCERT

49 Which of the following organizations has/have a bearing on the UN Climate Change’s Momentum for
Change initiative?
1. World Economic Forum (WEF)
2. Rockefeller Foundation
3. UNICEF

Select the correct answer using the codes below.


A. 1 only
B. 2 and 3 only
C. 1 and 2 only
D. 1, 2 and 3 Telegram Channel
https://t.me/visionpt3652019
Your Answer :
Correct Answer : C

Answer Justification :

Justification: Momentum for Change is an initiative spearheaded by the UN Climate Change


secretariat to shine a light on the enormous groundswell of activities underway across the globe
that are moving the world toward a highly resilient, low-carbon future.

The UN Climate Change’s Momentum for Change initiative is implemented with the support of The
Rockefeller Foundation. It operates in partnership with World Economic Forum (WEF), donors
supporting implementation of UN Climate Change’s Gender Action Plan and Climate Neutral Now.

Winners of the 2019 UN Global Climate Action Awards Announced.


Total Marks : 200
Online Prelims TEST - 12 (TEXTBOOK)
( InsightsIAS Mock Test Series for UPSC Preliminary Exam 2020 ) Mark Scored : 0

The 15 award-winning projects fall within four focus areas: Planetary Health, Climate
Neutral Now, Women for Results, and Financing for Climate Friendly Investment.

The Awards are spearheaded by the Momentum for Change initiative at UN Climate
Change.

From India, IT major Infosys has won the United Nations Global Climate Action Award
(UNGCAA) in the ‘Climate Neutral Now’ category for “Infosys’ Journey to Carbon Neutrality”.

Q Source:
https://www.insightsonindia.com/2019/09/28/insights-daily-current-affairs-pib-28-september-2019/

50 The Mauryan pillar capital symbolising Dhammachakrapravartana is found at

A. Sarnath
B. Vaishali
C. Nalanda
D. Rajgir

Your Answer :
Correct Answer : A

Answer Justification :

Learning: Found at Sarnath and popularly known as the Lion Capital is the finest example of
Mauryan sculptural tradition.

It is also our national emblem. It is carved with considerable care—voluminous roaring lion figures
firmly standing on a circular abacus which is carved with the figures of a horse, a bull, a lion and an
elephant in vigorous movement, executed with precision, showing considerable mastery in the
sculptural techniques.

This pillar capital symbolising Dhammachakrapravartana (the first sermon by the Buddha) has
become a standard symbol of this great historical event in the life of the

Q Source: Page 20: 11th NCERT: Introduction to Arts

51 “Super critical and ultra-super critical (USC) plants technologies” sometimes heard in news are
related to

A. Trade secrets negotiated under Nairobi package


B. Breeder’s rights under TRIPS
C. Reduction of atmospheric pollution
D. Fusion reactor technologies
Total Marks : 200
Online Prelims TEST - 12 (TEXTBOOK)
( InsightsIAS Mock Test Series for UPSC Preliminary Exam 2020 ) Mark Scored : 0

Your Answer :
Correct Answer : C

Answer Justification :

Learning: The concept of supercritical technology was introduced as a national plan in the
Integrated Energy Policy report of 2006 issued by the Planning Commission.

It noted that the average fuel conversion efficiency of Indian power plants is just about 30.5
per cent, though the new 500 MW plants have efficiency of 36 per cent. State-of-the-art
supercritical boilers can reach an efficiency level of 46 per cent depending on plant location.
Apart from being more efficient, these reactors also have capability to substantially reduce
CO2 emissions and virtually eliminate PM emissions.
It will help countries in reducing their emissions from coal, rather than reducing coal itself
and meet target provided in Intended Nationally Determined Contributions (INDCs).
Thus some target of Paris agreement on Climate Change about reducing the emissions from
coal power plants will be achieved.
It is evident that ultra-supercritical technology, where efficiencies greater than 45 per cent
can be achieved, is the next big change as coal-fired power plants enter a key stage of
evolution.

Q Source: Frequently in news

52 Consider the following statements.


Under the Indian Constitution
1. A person who voluntarily acquires citizenship of any other country is no longer an Indian citizen
2. An Overseas Citizen of India (OCI) loses his Indian citizenship if he violates any Indian law, whether
in domestic or foreign territory
3. Persons born outside of India shall not be considered citizens of India

Select the correct answer using the codes below.


A. 1 only
B. 2 only
C. 1 and 3 only
D. 3 only

Your Answer :
Correct Answer : A

Answer Justification :

Justification: Statement 1 and 3: Under Article 9 of the Indian Constitution, a person who
voluntarily acquires citizenship of any other country is no longer an Indian citizen.

In Section 8 of the Citizenship Act 1955, if an adult makes a declaration of renunciation of Indian
citizenship, he loses Indian citizenship.

From December 3, 2004, onwards, persons born outside of India shall not be considered citizens of
Total Marks : 200
Online Prelims TEST - 12 (TEXTBOOK)
( InsightsIAS Mock Test Series for UPSC Preliminary Exam 2020 ) Mark Scored : 0

India unless their birth is registered at an Indian consulate within one year of the date of birth.

Statement 2: An OCI is NOT a citizen of India, so 2 is wrong.

The Citizenship (Amendment) Bill, however, provides that the registration of Overseas Citizen of
India (OCI) cardholders may be cancelled if they violate any law. This will have a bearing on their
visa free access to the country.

Q Source:
https://www.insightsonindia.com/2019/09/28/insights-daily-current-affairs-pib-28-september-2019/

53 Which of the following conditions generally lead to coral bleaching?


1. Low sedimentation of ocean beds
2. Unusually warm ocean water
3. Acidification of oceans

Select the correct answer using the codes below.


A. 1 and 2 only
B. 2 only
C. 1 and 3 only
D. 2 and 3 only

Your Answer :
Correct Answer : D

Answer Justification :

Justification: Coral bleaching causes corals to expel tiny photosynthetic algae, draining them of
their colour. It calcifies the corals to turn into white. Algae are vital to the coral, which uses the
organic products of photosynthesis to help it grow. The loss of algae makes it vulnerable to disease
and it will eventually die. When a coral bleaches, it is not dead.

Conditions required for growth of corals are warm tropical oceans with minimum temperature of 20
degree; transparent parts of ocean bodies; Oceanic water must free of sedimentation (so, statement
1 is wrong); it should have relatively low salinity etc. When these conditions are vitiated it may lead
to coral bleaching.

Scientists have recorded worst mass coral bleaching event on Australia’s Great Barrier Reef
sounding the alarm over the delicate ecosystem.

Q Source: Recent Worst mass coral bleaching event on Australia’s Great Barrier Reef

54 With reference to National Water Mission, consider the following statements.


1. It is one of the eight missions launched under the National Action Plan on Climate Change
(NAPCC) for combating the threats of global warming.
2. It aims at increasing water use efficiency by 200% by 2025.
3. It aims at creating a comprehensive water data base in public domain for the assessment of the
Total Marks : 200
Online Prelims TEST - 12 (TEXTBOOK)
( InsightsIAS Mock Test Series for UPSC Preliminary Exam 2020 ) Mark Scored : 0

impact of climate change on water resources.

Select the correct answer using the codes below.


A. 1 and 3 only
B. 2 only
C. 1, 2 and 3
D. 2 and 3 only

Your Answer :
Correct Answer : A

Answer Justification :

Justification: It is one of the eight missions launched under the National Action Plan on Climate
Change (NAPCC) for combating the threats of global warming.

Under the mission, the National Water Policy would be revisited in consultation with States to
ensure basin level management strategies to deal with variability in rainfall and river flows due to
climate change.

Goals of the National Water Mission:

1. Comprehensive water data base in public domain and assessment of the impact of climate
change on water resource.

2. Promotion of citizen and state actions for water conservation, augmentation and preservation.

3. Focused attention to vulnerable areas including over-exploited areas.

4. Increasing water use efficiency by 20%.

5. Promotion of basin level integrated water resources management.

Objective of the National Water Mission:

1. To conserve water.

2. To minimise the wastage of the water.

3. To ensure equitable distribution across the country and within States through integrated
water resources management.
Total Marks : 200
Online Prelims TEST - 12 (TEXTBOOK)
( InsightsIAS Mock Test Series for UPSC Preliminary Exam 2020 ) Mark Scored : 0

Q Source: Frequently in news

55 Istanbul is located at the junction of

A. Black Sea and Mediterranean Sea


B. Red Sea and Caspian Sea
C. Caspian Sea and Black Sea
D. Mediterranean Sea and Caspian Sea

Your Answer :
Correct Answer : A

Answer Justification :

Learning: Istanbul is a transcontinental city in Eurasia, straddling the Bosphorus strait (which
separates Europe and Asia) between the Sea of Marmara and the Black Sea.

Istanbul's strategic position on the historic Silk Road, rail networks to Europe and the Middle East,
and the only sea route between the Black Sea and the Mediterranean have produced a cosmopolitan
populace.
Total Marks : 200
Online Prelims TEST - 12 (TEXTBOOK)
( InsightsIAS Mock Test Series for UPSC Preliminary Exam 2020 ) Mark Scored : 0

Q Source: Map-based questions: Middle-east

56 Consider the following statements.


1. One of the major reasons behind India’s low material productivity is that its resource extraction
(tonnes/acre) is significantly lower than the world average.
2. India is nearly 100% import dependent for the majority of the ‘most critical’ materials such as
cobalt, copper and lithium that find extensive application in high-end technology industry.

Select the correct answer using the codes below.


A. 1 only
B. 2 only
C. Both 1 and 2
D. None of the above

Your Answer :
Correct Answer : B

Answer Justification :

Justification: Challenges before India:

1. According to data available, India’s resource extraction of 1580 tonnes/acre is much higher
than the world average of 450 tonnes/acre, while material productivity remains low.

2. Water is fast becoming scarce while deteriorating air quality has emerged as a major threat to
human life.

3. There has been massive soil degradation, with 147 million hectares (Mha) of a total of 329
Mha land area hit.

4. Import dependency is nearly 100% for the majority of the ‘most critical’ materials -cobalt,
copper and lithium that find extensive application in high-end technology industry.

5. Over 80% of crude oil that is processed in the economy is imported, alongwith 85% of
its coking coal demand. Extraction of non-metallic minerals is crippled with challenges.

6. To add to the problems, the country’s recycling rate is just about 20-25% compared with 70%
in developing countries in Europe. The situation will only aggravate as India is likely to double
its material consumption by 2030.

Q Source:
https://www.insightsonindia.com/2019/09/25/insights-daily-current-affairs-pib-25-september-2019/
Total Marks : 200
Online Prelims TEST - 12 (TEXTBOOK)
( InsightsIAS Mock Test Series for UPSC Preliminary Exam 2020 ) Mark Scored : 0

57 Consider the following about the Press Council of India.


1. It is a quasi-judicial body.
2. It is headed by a retired judge of Supreme Court by convention.
3. It adjudicates the complaints made by the Indian Press.
4. Citizens can complain to this authority if the Press violates standard guidelines and engages in
unethical reporting.

Select the correct answer using the codes below.


A. 1, 2 and 4 only
B. 2 and 3 only
C. 1, 3 and 4 only
D. 1, 2, 3 and 4

Your Answer :
Correct Answer : D

Answer Justification :

Justification & Learning: It was first set up in 1966 by the Parliament on the recommendations of
the First Press Commission with the object of preserving the freedom of the press and of
maintaining and improving the standards of press in India.

The present Council functions under the Press Council Act 1978. It is a statutory, quasi judicial
body which acts as a watchdog of the press.

It adjudicates the complaints against and by the press for violation of ethics and for violation of the
freedom of the press respectively. So, both 3 and 4 are correct.

The Press Council is headed by a Chairman, who has by convention, been a retired judge of the
Supreme Court of India.

Q Source: Often in news due to Press reporting issues

58 The names of cyclone in the Bay of Bengal and Arabian Sea are chosen on the basis of

A. A pre-agreed naming system


B. The government of the country experiencing the highest intensity of the cyclone
C. The trade wind that causes the cyclone
D. The country experiencing the most damage from the cyclone

Your Answer :
Correct Answer : A

Answer Justification :

Justification: Each Tropical Cyclone basin in the world has its own rotating list of names.

For cyclones in the Bay of Bengal and Arabian Sea, the naming system was agreed by eight member
Total Marks : 200
Online Prelims TEST - 12 (TEXTBOOK)
( InsightsIAS Mock Test Series for UPSC Preliminary Exam 2020 ) Mark Scored : 0

countries of a group called WMO/ESCAP and took effect in 2004.

The member countries submitted eight names each. The first cyclone after the list was adopted was
given the name in the first row of the first column — Onil, proposed by Bangladesh.

Subsequent cyclones are being named sequentially, column-wise, with each cyclone given the name
immediately below that of the previous cyclone.

Once the bottom of the column is reached, the sequence moves to the top of the next column. So
far, the first seven columns have been exhausted.

Why name cyclones?

Makes it easier for the media to report on these cyclones, heightens interest in warnings, and
increases community preparedness.

Easier to remember than numbers and technical terms.

Q Source: Frequently in news

59 What do you understand by ‘protectionism’, a term often seen in newspapers?

A. Imposing trade restrictions to protect domestic industry


B. Entering into alliances with foreign militaries to safeguard one’s borders
C. Mandating military conscription for better defence capabilities
D. Protecting one’s citizen from influence of foreign cultures

Your Answer :
Correct Answer : A

Answer Justification :

Learning: In economics, protectionism is the economic policy of restraining trade between states
(countries) through methods such as tariffs on imported goods, restrictive quotas, and a variety of
other government regulations.

Protectionist policies protect the producers, businesses, and workers of the import-competing
sector in a country from foreign competitors.

However, they hurt consumers in general, and the producers and workers in export sectors, both in
the country implementing protectionist policies, and in the countries protected against.

Q Source: Frequently in news

60 World’s first Indian Partition Museum is situated in


Total Marks : 200
Online Prelims TEST - 12 (TEXTBOOK)
( InsightsIAS Mock Test Series for UPSC Preliminary Exam 2020 ) Mark Scored : 0

A. Amritsar
B. New Delhi
C. Kolkata
D. Lahore

Your Answer :
Correct Answer : A

Answer Justification :

Learning: The Museum is part of the newly inaugurated Heritage Street at Amritsar, which starts
at the Golden Temple and ends at the Town Hall.

The Partition Museum Project (TPMP) was initiated by The Arts and Cultural Heritage Trust
(TAACHT) to work towards the establishment of a world class, physical museum, dedicated to the
memory of the Partition of the sub-continent in 1947 — its victims, its survivors and its lasting
legacy.

Q Source: http://www.partitionmuseum.org/about-us/

61 The education conference held at Wardha in October 1937 is notable for


1. Promoting free and compulsory education for an individual throughout his life
2. Endorsing Gandhiji's proposals for 'basic education' through the vernacular medium and manual
productive work
3. Motivating teachers to work without any payment for the welfare of nation at large

Select the correct answer using the codes below.


A. 1 and 2 only
B. 2 only
C. 2 and 3 only
D. 1 and 3 only

Your Answer :
Correct Answer : B

Answer Justification :

Justification: In 1937, Gandhiji had published an article in the Harijan, based upon which, an all
India National Education Conference was held in 1937 - Wardha Educational Conference. The
president of this conference was Gandhi.

The resolutions passed were as follows:

Free and compulsory education to be provided for 7 years at a nationwide scale.


Mother tongue should be the medium of instruction.
Throughout this period of 7 years, the education should be around some forms of manual and
productive work and for this purpose a handicraft must be chosen, based upon the
environment of the child.
Total Marks : 200
Online Prelims TEST - 12 (TEXTBOOK)
( InsightsIAS Mock Test Series for UPSC Preliminary Exam 2020 ) Mark Scored : 0

This system would generate the remuneration of the teachers. So, 3 is wrong.

Also, a Basic Education Committee was setup under Dr. Zakir Hussain following Wardha conference
to formulate the scheme of the basic education.

The aim of the basic education was to develop the qualities of the ideal citizenship and more aspect
should be give to the Indian culture than the literacy. There should be NO PLACE for English in the
curriculum. There was no place for religious education in this scheme.

Q Source: Improvisation: Past year UPSC papers

62 ‘Bharat 22’, sometimes seen in news, aims to

A. Bolster Government’s disinvestment programme


B. Improve conservation of the 22 scheduled languages in the Constitution of India
C. Increase maritime and land security of all coastal and border states of India
D. Reduce red tapism in the delivery of welfare schemes

Your Answer :
Correct Answer : A

Answer Justification :

Learning: This is an exchange traded fund (ETF) launched by Union Finance Ministry.

It will help to speed up disinvestment programme budgeted to raise a record Rs 72,500 crore
in the FY 2018.
Bharat 22 spans six sectors — basic materials, energy, finance, industrials, FMCG and
utilities.
The sector wise weightage in the Bharat 22 Index is basic materials (4.4%), energy (17.5%),
finance (20.3%), FMCG (15.2%), industrials (22.6%), and utilities (20%).

Q Source: Revision: Current Affairs (last 2 years)


http://economictimes.indiatimes.com/markets/stocks/news/reasons-why-bharat-22-etf-can-be-a-win-
win-deal-for-you/articleshow/60029997.cms

63 Consider the following statements.


1. The first demand for a Constituent Assembly was made right after the Non-cooperation movement
was withdrawn.
2. The Constituent assembly was constituted in 1943 after the approval of the Cripps Proposals.

Which of the above is/are correct?


A. 1 only
B. 2 only
C. Both 1 and 2
D. None
Total Marks : 200
Online Prelims TEST - 12 (TEXTBOOK)
( InsightsIAS Mock Test Series for UPSC Preliminary Exam 2020 ) Mark Scored : 0

Your Answer :
Correct Answer : D

Answer Justification :

Justification: MN Roy had put forth the idea of a Constituent assembly of India in 1934. Later the
INC demanded it.

During the Second World War, this assertion for an independent Constituent Assembly formed only
of Indians gained momentum and this was convened in December 1946.

Between December 1946 and November 1949, the Constituent Assembly drafted a constitution for
independent India.

Q Source: Page 5: 8th NCERT Social and Political Life

64 The NOTA voting option is NOT applicable in

A. Lok Sabha elections


B. State assembly elections
C. Election of the President and Vice-President and in Rajya Sabha elections
D. All of the above

Your Answer :
Correct Answer : C

Answer Justification :

Learning: Option C: The option of NOTA would be against the dignity of the election of Presidential
office.

The Election Commission, in 2013, issued directions for providing the NOTA option in elections. But
then, doubts were raised about its applicability in the Rajya Sabha polls.

After examining the issue, the EC in 2014, directed that the option would also apply for elections to
the Rajya Sabha.

Recently, after a petition was filed,

A Bench, led by Chief Justice of India (CJI) Dipak Misra held that the NOTA option is meant only for
universal adult suffrage and direct elections and not polls held by the system of proportional
representation by means of the single transferable vote as done in the Rajya Sabha.

Q Source: Revision: Current Affairs: Last 2 years

65 In order to stay over the same location on the Earth, a geostationary satellite must be be directly
above the
Total Marks : 200
Online Prelims TEST - 12 (TEXTBOOK)
( InsightsIAS Mock Test Series for UPSC Preliminary Exam 2020 ) Mark Scored : 0

A. Tropic of Cancer
B. Either North or South Pole
C. Equator
D. Tropic of Capricon

Your Answer :
Correct Answer : C

Answer Justification :

Justification & Learning: Most of the communication satellites today are placed in a
geostationary orbit.

Geostationary satellites in orbits circle the Earth at the same rate as the Earth spins.

The Earth takes nearly 24 hours to make one full revolution around the earth allowing them to
observe a location constantly on earth since earth too rotates at the same speed. So based on
Kepler's Laws of Planetary Motion, this would put the satellite at approximately 35,790 km
above the Earth.

The satellites are located near the equator since at this latitude there is a constant force of
gravity from all directions. At other latitudes, the bulge at the centre of the Earth would pull
the satellite down.

Q Source: 12th Physics NCERT

66 Lithospheric plates move around very slowly – just a few millimetres each year. What is the reason
behind this movement?

A. Ocean currents
B. Movement of magma inside earth
C. Formation of folds on earth’s crust
D. Rotation of earth

Your Answer :
Correct Answer : B

Answer Justification :

Learning: The lithosphere is broken into a number of plates known as the Lithospheric plates.

The molten magma inside the earth moves in a circular manner. These plates move because of the
movement of the molten magma inside the earth.

Lateral movements between lithospheric plates create transform faults at the sites of plate slippage.
Total Marks : 200
Online Prelims TEST - 12 (TEXTBOOK)
( InsightsIAS Mock Test Series for UPSC Preliminary Exam 2020 ) Mark Scored : 0

Q Source: 7th NCERT Geography

67 A Communist economy advocates


1. State ownership of all properties including labour in the economy
2. Absolute power to state in running and managing the economy

Which of the above is/are correct?


A. 1 only
B. 2 only
C. Both 1 and 2
D. None

Your Answer :
Correct Answer : C

Answer Justification :

Justification: Communism is an economic system where the collective owns the factors of
production. The four factors of production are labor, entrepreneurship, capital goods and natural
resources.

The following form major features of the communist economies:

Abolition of property in land and application of all rents of land to public purposes.
A heavy progressive or graduated income tax.
Abolition of all right of inheritance.

Q Source: Revision

68 Bhimbetka caves shows the continuity of human evolution from the Lower Palaeolithic Period to the
Mesolithic period to the chalcolithic period. Why?
1. It contains evidence of the use of tools and implements from all these periods.
2. There are a large number of Shanka Lipi inscriptions in the Bhimbetka cluster of rock shelters.

Which of the above is/are correct?


A. 1 only
B. 2 only
C. Both 1 and 2
D. None

Your Answer :
Correct Answer : A

Answer Justification :

Justification: Statement 1: In the caves, the continuity of human evolution from the Lower
Palaeolithic Period is noticed by the smaller size of stone tools in the following Middle Palaeolithic
Total Marks : 200
Online Prelims TEST - 12 (TEXTBOOK)
( InsightsIAS Mock Test Series for UPSC Preliminary Exam 2020 ) Mark Scored : 0

Period besides new tools like scrapers.

During the Upper Palaeolithic Period newer tool types like: blades, borers and burins had also
emerged.
However, it is in the Mesolithic Period that there is a clear change in the materials and tool
typology.
Earlier, the tools were largely made of quartzite and sandstone, whereas the tools being made
in the Mesolithic Period were most often of chalcedony.
The Mesolithic culture at Bhimbetka continued much longer as understood by the presence of
Chalcolithic potteries in otherwise Mesolithic contexts.
By the Early Historic times it appears that interaction with the surrounding cultures became
more pronounced.
This is evidenced by the presence of rock-cut beds in a rock shelter on the top portion of an
inselberg like outcrop not far from the later built temple at this site.

Statement 2: Shankhalipi or "shell-script" is a term used by scholars to describe ornate spiral


Brahmi characters that resemble conch shells (or shankhas). They are found in inscriptions across
various parts of India except the far south and date to between the 4th and 8th centuries CE.

http://asi.nic.in/asi_monu_whs_rockart_bhimbetka.asp

Q Source: Additional Research: Page 3: 11th NCERT: Introduction to Arts

69 Which of the following land arrangements were prevalent after the application of permanent
settlement in Rural Bengal?
1. Zamindars were responsible for paying revenue to the company and distributing the revenue
demand over villages.
2. Each village ryot paid rent to the zamindar.
3. Jotedars gave out loans to ryots and sold their produce.

Select the correct answer using the codes below.


A. 1 and 2 only
B. 2 and 3 only
C. 3 only
D. 1, 2 and 3

Your Answer :
Correct Answer : D

Answer Justification :

Background and Justification: Earlier zamindars in Bengal, Bihar and Odisha had been
functionaries who held the right to collect revenue on behalf of the Mughal emperor and his
representative, the diwan, in Bengal.

The diwan supervised the zamindars to ensure they were neither lax nor overly stringent.
Total Marks : 200
Online Prelims TEST - 12 (TEXTBOOK)
( InsightsIAS Mock Test Series for UPSC Preliminary Exam 2020 ) Mark Scored : 0

When the East India Company was awarded the diwani or overlordship of Bengal they
changed the system by giving zamindars more power in certain areas and assuming the
ownership of land by themselves.

Statement 3: Jotedars were powerful people who were often rich ryots. Next in hierarchy were
Ryots who cultivated some land and gave out the rest to under-ryots. Under-ryots paid rent to the
ryots.

The image shows the distribution of Power in rural Bengal of that time:

Q Source: 12th NCERT History: Part-III

70 Todas and Moplahs tribes can be found in the states of

A. Rajasthan and Nagaland


B. Tamil Nadu and Kerala
C. West Bengal and Odisha
D. Andhra Pradesh and Tamil Nadu

Your Answer :
Correct Answer : B

Answer Justification :

Learning: The Toda people are a small pastoral tribal community who live on the isolated Nilgiri
plateau in hill country of Southern India.

Since the early 21st century, Toda society and culture have been the focus of an international
effort at culturally sensitive environmental restoration.
The Toda lands are now a part of the Nilgiri Biosphere Reserve, a UNESCO-designated
Total Marks : 200
Online Prelims TEST - 12 (TEXTBOOK)
( InsightsIAS Mock Test Series for UPSC Preliminary Exam 2020 ) Mark Scored : 0

International Biosphere Reserve; their territory is declared UNESCO World Heritage Site.
The Moplah peasant movement was engineered in August 1921 among the peasants of
Malabar district in Kerala. The Moplah tenants were Muslims and they agitated against the
Hindu landlords and the British government.
Their grievances related to lack of any security of tenure, renewal fees, high rents and other
oppressive landlord exactions. In the 19th century as well, there had been cases of Moplah
resistance to landlord oppression but what erupted in 1921 was on a different scale
altogether.

Q Source: Tribes of India

71 Most of the states with less than 10 per cent of the forest area lie in which part of the country?

A. North and north-western regions


B. Eastern belts
C. Southern and south-western belt
D. South-eastern region

Your Answer :
Correct Answer : A

Answer Justification :

Justification and Learning: These are Rajasthan, Gujarat, Punjab, Haryana and Delhi.

There is a lot of variation in actual forest cover, which ranges from 9.56 per cent in Jammu
and Kashmir to 84.01 per cent in Andaman and Nicobar Islands.
According to state records, the forest area covers 23.28 per cent of the total land area of the
country.
It is important to note that the forest area and the actual forest cover are not the same.
The forest area is the area notified and recorded as the forest land irrespective of the
existence of trees, while the actual forest cover is the area occupied by forests with canopy.
The former is based on the records of the State Revenue Department, while the latter is based
on aerial photographs and satellite imageries.

Most of the forests in Punjab and Haryana have been cleared for cultivation. States with 10-20 per
cent forest area are Tamil Nadu and West Bengal. In Peninsular India, excluding Tamil Nadu, Dadra
and Nagar Haveli and Goa, the area under forest cover is 20-30 per cent. The north-eastern states
have more than 30 per cent of the land under forest.

Hilly topography and heavy rainfall are good for forest growth.
Total Marks : 200
Online Prelims TEST - 12 (TEXTBOOK)
( InsightsIAS Mock Test Series for UPSC Preliminary Exam 2020 ) Mark Scored : 0

Q Source: Page 61: 11th NCERT: India Physical Geography

72 Among the following, the cave site that has Buddhist, Brahmanical and Jain works is?

A. Ellora
B. Ajanta
C. Kanheri
D. Badami

Your Answer :
Correct Answer : A

Answer Justification :
Total Marks : 200
Online Prelims TEST - 12 (TEXTBOOK)
( InsightsIAS Mock Test Series for UPSC Preliminary Exam 2020 ) Mark Scored : 0

Learning: It is located in Aurangabad District is Ellora. It is located a hundred kilometres from


Ajanta and has thirty-two Buddhist, Brahmanical and Jain caves.

Locally known as ‘Verul Leni’, the caves are hewn out of the volcanic basaltic formation of
Maharasthra, known as ‘Deccan Trap’,

Ellora is also world famous for the largest single monolithic excavation in the world, the great
Kailasa.

It is a unique art-historical site in the country as it has monastries associated with the three
religions dating from the fifth century CE onwards to the eleventh century CE.

Learning: The importance of Ellora during the early centuries of the Christian era is also
understood by the findings of coins of Satavahanas, the ruling dynasty during the period. The
Satavahanas had their capital at Pratishtana (modern Paithan) and ruled the entire area between
the Arabian Sea and Bay of Bengal and bordered by the Narmada River on the north.

Ellora is located directly on the ancient trade route which traversed from Pratishtana via
Aurangabad, Ellora, Pital Khora, Patne, Nasika (modern Nasik). Nasik is at the crossroads of an
ancient trade route connecting centres on the west to east and those on the north to south.

Q Source: Page 36: 11th NCERT: Introduction to Arts

http://asi.nic.in/asi_monu_whs_ellora.asp

73 EI-Nino is a complex weather system that appears once every three to seven years. In the Southern and Central
Pacific Ocean, it results in
#65212
Total Marks : 200
Online Prelims TEST - 12 (TEXTBOOK)
( InsightsIAS Mock Test Series for UPSC Preliminary Exam 2020 ) Mark Scored : 0

1. Distortion of equatorial atmospheric circulation


2. Irregularities in the evaporation of sea water
3. Increase in the amount of diatoms in the ocean

Select the correct answer using the codes below.


A. 1 and 2 only
B. 3 only
C. 1 and 3 only
D. 1, 2 and 3

Your Answer :
Correct Answer : A

Answer Justification :

Justification: Statement 1 and 2: EI-Nino is merely an extension of the warm equatorial current
which gets replaced temporarily by cold Peruvian current or Humbolt current (locate these currents
in your atlas).

The system involves oceanic and atmospheric phenomena with the appearance of warm
currents off the coast of Peru in the Eastern Pacific and affects weather in many places
including India.
This current increases the temperature of water on the Peruvian coast by 10°C. This affects
the local temperature, and thus ocean currents, wind pattern leading to rainfall changes.
It brings drought, floods and other weather extremes to different parts of the world.
EI-Nino is used in India for forecasting long range monsoon rainfall.

Statement 3: It actually reduces the amount of phytoplankton and also diatoms because of the
increase in the sea temperature. Planktons need a more ambient climate to survive.

Q Source: Page 46: 11th NCERT: India Physical Geography

74 What is the advantage of using Proton-exchange membrane fuel cells (PEMFCs) over ordinary fuel
cells like solid oxide fuel cells (SOFCs)?
1. PEMs operate at extremely high temperatures making them ideal for high combustion processes
such as those occurring in a blast furnace.
2. They have a rapid start-up time as compared to ordinary fuel cells.

Select the correct answer using the codes below.


A. 1 only
B. 2 only
C. Both 1 and 2
D. None of the above

Your Answer :
Correct Answer : B

Answer Justification :
Total Marks : 200
Online Prelims TEST - 12 (TEXTBOOK)
( InsightsIAS Mock Test Series for UPSC Preliminary Exam 2020 ) Mark Scored : 0

Concept: A fuel cell is an electrochemical cell that converts the chemical energy of a fuel (often
hydrogen) and an oxidizing agent (often oxygen) into electricity through a pair of redox reactions.

Fuel cells are different from most batteries in requiring a continuous source of fuel and oxygen
(usually from air) to sustain the chemical reaction, whereas in a battery the chemical energy usually
comes from metals and their ions or oxides that are commonly already present in the battery,
except in flow batteries. Fuel cells can produce electricity continuously for as long as fuel and
oxygen are supplied.

Justification: Proton-exchange membrane fuel cells (PEMFCs) are believed to be the most
promising type of fuel cell to act as the vehicular power source replacement for gasoline and diesel
internal combustion engines.

They are being considered for automobile applications because they typically have a low operating
temperature (~80 °C) and a rapid start-up time, including from frozen conditions. PEMFCs operate
at 40–60% efficiency and can vary the output to match the demands.

Statement 1 is incorrect since a blast furnace has temperatures ranging way above 700-800 degree
Celsius upto over 2000 degrees.

First used in the 1960s for the NASA Gemini program, PEMFCs are currently being developed and
demonstrated from ~100 kW cars to a 59 MW power plant.[citation needed]

PEMFCs contain advantages over other types of fuel cells such as solid oxide fuel cells (SOFC).
PEMFCs operate at a lower temperature, are lighter and more compact, which makes them ideal for
applications such as cars.

Q Source:
https://www.insightsonindia.com/2019/09/27/insights-daily-current-affairs-pib-27-september-2019/

75 Consider the following statements.


1. Hair washed with hard water looks dull because of presence of calcium and magnesium ions in hard
water.
2. Dye absorb evenly on clothes washed with soap in hard water due to the formation of the gummy
mass of potassium salts.

Which of the above is/are correct?


A. 1 only
B. 2 only
C. Both 1 and 2
D. None

Your Answer :
Correct Answer : A

Answer Justification :

Justification: Hard water contains calcium and magnesium ions. These ions form insoluble calcium
Total Marks : 200
Online Prelims TEST - 12 (TEXTBOOK)
( InsightsIAS Mock Test Series for UPSC Preliminary Exam 2020 ) Mark Scored : 0

and magnesium soaps respectively when sodium or potassium soaps are dissolved in hard water.

These insoluble soaps separate as scum in water and are useless as cleansing agent. In fact these
are hinderance to good washing, because the precipitate adheres onto the fibre of the cloth as
gummy mass. Hair washed with hard water looks dull because of this sticky precipitate.

Dye does not absorb evenly on cloth washed with soap using hard water, because of this gummy
mass.

Q Source: 12th NCERT: Environmental Chemistry

76 Littoral and swamp forests are most likely to be found in

A. Eastern Coastal belts


B. Central India
C. Western Ghats
D. North-eastern India

Your Answer :
Correct Answer : A

Answer Justification :

Justification: Littoral and swamp forests are some of the most important wetlands of India.

India has a rich variety of wetland habitats. About 70 per cent of this comprises areas under paddy
cultivation.

The total area of wet land is 3.9 million hectares.


Total Marks : 200
Online Prelims TEST - 12 (TEXTBOOK)
( InsightsIAS Mock Test Series for UPSC Preliminary Exam 2020 ) Mark Scored : 0

Learning: The country’s wetlands have been grouped into eight categories, viz.

the reservoirs of the Deccan Plateau in the south together with the lagoons and other
wetlands of the southern west coast;
the vast saline expanses of Rajasthan, Gujarat and the Gulf of Kachchh;
freshwater lakes and reservoirs from Gujarat eastwards through Rajasthan (Keoladeo
National Park) and Madhya Pradesh;
the delta wetlands and lagoons of India’s east coast (Chilika Lake); (v) the freshwater marshes
of the Gangetic Plain;
the floodplains of the Brahmaputra; the marshes and swamps in the hills of northeast India
and the Himalayan foothills;
the lakes and rivers of the montane region of Kashmir and Ladakh; and
the mangrove forest and other wetlands of the island arcs of the Andaman and Nicobar
Islands

Two sites — Chilika Lake (Orissa) and Keoladeo National Park (Bharatpur) are protected as water-
fowl habitats under the Convention of Wetlands of International Importance (Ramsar Convention).
Total Marks : 200
Online Prelims TEST - 12 (TEXTBOOK)
( InsightsIAS Mock Test Series for UPSC Preliminary Exam 2020 ) Mark Scored : 0

Q Source: Page 58: 11th NCERT: India Physical Geography

77 Places in the mountains are usually cooler than places on the plains due to the presence of

A. Leeward wind flow on the mountains


B. Moisture-laden winds on the mountains
C. Thin air on the mountains
D. All of the above

Your Answer :
Correct Answer : C

Answer Justification :

Justification: Density of air reduces with altitude, and so it is its ability to transfer the heat that
the earth receives from the Sun.

You know that the atmosphere is not directly heated by the Sun, but by the air that is in contact
with the earth.

So, thin air reduces the warming potential in the mountains. Temperatures decline with gain in
altitude.

For example, Agra and Darjiling are located on the same latitude, but temperature of January in
Agra is 16°C whereas it is only 4°C in Darjiling.

Q Source: Page 34: 11th NCERT: India Physical Geography

78 With respect to the United Nations General Assembly (UNGA), consider the following statements.
1. Each sovereign state gets one vote and the votes are not binding on the membership, except in
budgetary matters.
2. The UNGA decides on matters like the admission of new members to the UN

Select the correct answer using the codes below.


A. 1 only
B. 2 only
C. Both 1 and 2
D. None of the above

Your Answer :
Correct Answer : C

Answer Justification :

Justification: Popularly known as the parliament of the world, where all the 193 UN member
states are represented, the UNGA is the deliberative, policymaking and representative organ of the
UN.
Total Marks : 200
Online Prelims TEST - 12 (TEXTBOOK)
( InsightsIAS Mock Test Series for UPSC Preliminary Exam 2020 ) Mark Scored : 0

Roles and functions:

Takes a decision on important matters such as peace and security, discusses various global
issues and budgetary matters.
Decides on matters such as the admission of new members.
Decisions are taken through a vote. Admission of new members and budgetary matters
require a two-thirds majority, while the decision on other issues are taken by a simple
majority.
Each sovereign state gets one vote and the votes are not binding on the membership, except
in budgetary matters.
The Assembly has no binding votes or veto powers like the UN Security Council.
The UNGA can express world opinion, promote international cooperation in various fields and
make recommendations to the UNSC and elect the Security Council’s non-permanent
members.

Q Source:
https://www.insightsonindia.com/2019/09/28/insights-daily-current-affairs-pib-28-september-2019/

79 Which of these European nations lie completely to the north of Turkey?


1. Spain
2. Greece
3. Germany
4. Ukraine
5. Italy

Select the correct answer using the codes below.


A. 1, 3, 4 and 5 only
B. 3 and 4 only
C. 2, 3 and 4 only
D. 1, 2 and 5 only

Your Answer :
Correct Answer : B

Answer Justification :
Total Marks : 200
Online Prelims TEST - 12 (TEXTBOOK)
( InsightsIAS Mock Test Series for UPSC Preliminary Exam 2020 ) Mark Scored : 0

Q Source: Map based questions: Europe

80 FOREST PLUS 2.0, recently seen in news, is a joint initiative of


1. India’s Ministry of Environment, Forest and Climate Change (MoEF&CC)
2. United Nations Environment Programme (UNEP) – Department of Conservation (DoC)
3. REDD++ framework

Select the correct answer using the codes below.


A. 1 and 2 only
B. 2 and 3 only
C. 1 and 4 only
D. 3 and 4 only

Your Answer :
Correct Answer : C

Answer Justification :

Justification: US Agency for International Development (USAID) and India’s Ministry of


Environment, Forest and Climate Change (MoEF&CC) have launched Forest-PLUS 2.0.
Total Marks : 200
Online Prelims TEST - 12 (TEXTBOOK)
( InsightsIAS Mock Test Series for UPSC Preliminary Exam 2020 ) Mark Scored : 0

1. Forest-PLUS is a five-year programme initiated in December 2018 that focuses on developing


tools and techniques to bolster ecosystem management and harnessing ecosystem services in
forest landscape management.

2. Forest-PLUS 2.0, the second set of pilot projects, is meant to enhance sustainable forest
landscape management after Forest-PLUS completed its five years in 2017.

3. The programme’s first set focused on capacity building to help India participate in Reducing
Emissions from Deforestation and forest Degradation (REDD+). It included four pilot projects
in Sikkim, Rampur, Shivamogga and Hoshangabad.

4. Under these, field tests, innovative tools and approaches for Indian forest management were
developed. Promotion of bio-briquettes in Sikkim, introduction of solar heating systems in
Rampur and development of an agro-forestry model in Hoshangabad were some of the
achievements of this programme.

5. Forest-PLUS 2.0 comprises pilot project in three landscapes — Gaya in Bihar,


Thiruvananthapuram in Kerala and Medak in Telangana. The choice of these sites was driven
by the contrast in their landscapes – Bihar is a forest deficit area, Telangana is a relatively
drier area where there is ample scope for community livelihood enhancement and Kerala is
rich in biodiversity.

Q Source:
https://www.insightsonindia.com/2019/09/27/insights-daily-current-affairs-pib-27-september-2019/

81 The Ancient Silk Route traversed through which of the following present-day regions?
1. Europe
2. Africa
3. East Asia
4. South-east Asia

Select the correct answer using the codes below.


A. 1 and 2 only
B. 1, 3 and 4 only
C. 2 and 4 only
D. 1, 2, 3 and 4

Your Answer :
Correct Answer : D

Answer Justification :

Justification: The road connected civilizations across Europe, Asia (including Persia, Arabia) and
Africa (only the horn part).
Total Marks : 200
Online Prelims TEST - 12 (TEXTBOOK)
( InsightsIAS Mock Test Series for UPSC Preliminary Exam 2020 ) Mark Scored : 0

The disappearance of the Silk Road following the end of the Mongols' reign was one of the main
factors that stimulated the Europeans to reach the prosperous Chinese empire through another
route, especially by sea.

The direct ocean route from Europe to the East was finally opened by the expeditions of Bartolomeu
Dias (1488), and Vasco da Gama (1497-1499), by the Atlantic and the Indian oceans.

597
07
044
89
.in-
rs ity
e
niv
is tu
.chr
e
e rc
m
om
z @c
n de
rna
n .fe
o
Q Source: 10th NCERTrWorld History
ha
z -s
82 International de
nOrganisations that India is a member of include
r na
e
1. Commonwealth of Nations
2.ro n Fof Bengal Initiative for Multi-Sectoral Technical and Economic Cooperation (BIMSTEC)
Bay
a
Sh3. East Asia Summit

Select the correct answer using the codes below.


A. 1 only
B. 1 and 2 only
C. 2 and 3 only
D. 1, 2 and 3

Your Answer :
Correct Answer : D

Answer Justification :

Justification: India is a member of SAARC, Commonwealth of Nations, G20, EAS, UN, BRICS,
G8+5, BIMSTEC, Mekong-Ganga Cooperation, NAM, Indian Ocean Rim Association, IBSA, SCO
Total Marks : 200
Online Prelims TEST - 12 (TEXTBOOK)
( InsightsIAS Mock Test Series for UPSC Preliminary Exam 2020 ) Mark Scored : 0

(formalizing), MTCR.

If you find some other forum of which India is a part of, please drop a comment in the test
discussion page.

Statement 1: It is an intergovernmental organisation of 52 member states that are mostly former


territories of the British Empire.

The Commonwealth operates by intergovernmental consensus of the member states,


organised through the Commonwealth Secretariat and non-governmental organisations.
Member states have no legal obligation to one another. Instead, they are united by language,
history, culture and their shared values of democracy, free speech, human rights, and the rule
of law.

Statement 2: Members of BIMSTEC are: Bangladesh, India, Myanmar, Sri Lanka, Thailand, Bhutan
and Nepal. The BIMSTEC states are among the countries dependent on the Bay of Bengal. It
cooperates over a variety of issues from trade to agriculture.

Statement 3: The East Asia Summit (EAS) is a forum held annually by leaders of, initially, 16
countries in the East Asian, Southeast Asian and South Asian regions. Membership expanded to 18
countries including the United States and Russia at the Sixth EAS in 2011.

Q Source: International Organizations

83 What is the constitutional provision regarding the removal of an Election Commissioner?

A. On an impeachment motion passed by the Cabinet


B. On the directions of the Supreme Court
C. On an office order of the Chief Election Commissioner
D. None of the above

Your Answer :
Correct Answer : D

Answer Justification :

Justification: The CEC and the Election Commissioners have a tenure of six years, or up to the age
of 65, whichever is earlier, and enjoy the same status and receive salary and perks as available to
Supreme Court judges.

The CEC and the Election Commissioners enjoy the same decision-making powers which is
suggestive of the fact that their powers are at par with each other.

Though the CEC is provided with a security of tenure and could only be removed through
impeachment, other EC’s can be removed on the recommendations of CEC to President.

However, Article 324(5) does not provide similar protection to the Election Commissioners and it
merely says that they cannot be removed from office except on the recommendation of the CEC.
Total Marks : 200
Online Prelims TEST - 12 (TEXTBOOK)
( InsightsIAS Mock Test Series for UPSC Preliminary Exam 2020 ) Mark Scored : 0

There was a petition filed on this issue in the SC to seek greater clarity on what exact the
recommendation of CEC mean in this context. Is the removal process same as that of the CEC
because they enjoy nearly the same powers; or is it different?

Q Source:
http://www.insightsonindia.com/2017/12/02/insights-daily-current-affairs-02-december-2017/

84 Which of the following are the major economic challenges for a State Economy?
1. Deciding what goods to produce
2. Ensuring a balance between supply and demand of goods
3. To ascertain the level of investment required for an optimal productive capacity in the economy

Select the correct answer using the codes below.


A. 1, 2 and 3
B. 1 and 3 only
C. 2 only
D. 2 and 3 only

Your Answer :
Correct Answer : A

Answer Justification :

Justification: Every economy meets with certain challenge.

One, to ascertain the availability of the goods and services required by the population and second,
the presence of the supply network.

Every economy has to, at first, guarantee the required level of goods and services out of its
production process.

For this, proper level of production capacity should be built which requires a particular level of
capital formation or investment. From where the investible funds will be managed is altogether a
separate question.

Whether the investment will come from the government, the domestic private sector or the
foreigners?

Once these details are cleared and selected as per the socio-economic condition of the economy, a
proper distribution network for goods and services produced is assured.

Statement 3: But, in a State Economy the private sector does not play any role and is nationalized
by the government.

Q Source: 11th NCERT: Economics

85 How do tropical cyclones play an important role in modulating regional and global climate?
1. They carry heat energy away from the tropics and transport it toward temperate latitudes.
Total Marks : 200
Online Prelims TEST - 12 (TEXTBOOK)
( InsightsIAS Mock Test Series for UPSC Preliminary Exam 2020 ) Mark Scored : 0

2. They can often bring intense rainfall to drought prone regions.

Which of the above is/are correct?


A. 1 only
B. 2 only
C. Both 1 and 2
D. None

Your Answer :
Correct Answer : C

Answer Justification :

Justification: Statement 2: Tropical cyclones can relieve drought conditions as they often carry
moisture laden winds.

In addition to strong winds and rain, tropical cyclones are capable of generating high waves,
damaging storm surge, and tornadoes.

They typically weaken rapidly over and where they are cut off from their primary energy source.
For this reason, coastal regions are particularly vulnerable to damage from a tropical cyclone as
compared to inland regions.

Heavy rains. however, can cause significant flooding inland, and storm surges can produce
extensive coastal flooding up to 25 miles from the coastline.

Statement 1: They also carry heat energy away from the tropics and transport it toward temperate
latitudes, because they are large wind systems that absorb the frontal conditions of the area they
flow in.

Finally, due to the high speed they blow in, they also act as the earth's filter system by cleansing the
air of toxins and pollutants out of the atmosphere.

Q Source: Page 35: 11th NCERT: India Physical Geography

86 CARICOM is a group of countries situated in the

A. South Pacific Ocean


B. Atlantic Ocean
C. Indian Ocean
D. Arabian Sea

Your Answer :
Correct Answer : B

Answer Justification :

Justification: It is an organisation of fifteen Caribbean nations (members located mainly in the


Total Marks : 200
Online Prelims TEST - 12 (TEXTBOOK)
( InsightsIAS Mock Test Series for UPSC Preliminary Exam 2020 ) Mark Scored : 0

North Atlantic Ocean) and dependencies having primary objectives to promote economic
integration and cooperation among its members, to ensure that the benefits of integration are
equitably shared, and to coordinate foreign policy.

The organisation was established in 1973.

Its major activities involve coordinating economic policies and development planning;
devising and instituting special projects for the less-developed countries within its
jurisdiction; operating as a regional single market for many of its members (Caricom Single
Market); and handling regional trade disputes. The secretariat headquarters is
in Georgetown, Guyana.

CARICOM is an official United Nations Observer.

Q Source:
https://www.insightsonindia.com/2019/09/27/insights-daily-current-affairs-pib-27-september-2019/

87 Agro-forestry is

A. Altering the agriculture land use category to forest category


B. Clearing forests for slash and burn agriculture
C. Raising of trees and agriculture crops on the same land inclusive of the waste patches
D. Harnessing the practices of local tribal communities in raising crops inside a forest
Total Marks : 200
Online Prelims TEST - 12 (TEXTBOOK)
( InsightsIAS Mock Test Series for UPSC Preliminary Exam 2020 ) Mark Scored : 0

Your Answer :
Correct Answer : C

Answer Justification :

Learning: It combines forestry with agriculture, thus, altering the simultaneous production of food,
fodder, fuel, timber and fruit.

Community forestry involves the raising of trees on public or community land such as the village
pasture and temple land, roadside, canal bank, strips along railway lines, and schools etc.

It maintains or increases total yields by combining food crops (annuals) with tree crops (perennials)
and/or livestock on the same unit of land, either alternately or at the same time, using management
practices that suit the social and cultural characteristics of the local people and the economic and
ecological conditions of the area.

Q Source: Chapter 6: 11th NCERT: Physical Geography

88 Which among the following is NOT a form of Indian martial arts?

A. Kalarippayattu
B. Silambam
C. Chhau
D. Sattriya

Your Answer :
Correct Answer : D

Answer Justification :

Justification: Option A: It is a martial art, which originated as a style in Kerala, southern India
(North Malabar). The word kalari first appears in the Tamil Sangam literature (c. 300 BCE to 300
CE) to describe both a battlefield and combat arena.

Each warrior in the Sangam era received regular military training. It is considered to be one of the
oldest surviving fighting systems still in existence in the world

Option B: It is a weapon-based Martial Art of India, more specifically in the state of Tamil Nadu. The
word “Silambam” means either a mountain or merely to sound. While the Silambam fencers are
fighting, the weapon makes sound. This might have been the cause for its being named as
“Silambam”.

Option C: Chhau, is a semi classical Indian dance with martial, tribal and folk origins. with origins in
the eastern Indian states of Odisha, Jharkhand and West Bengal.

Option D: Sattriya Nritya, is a major Indian classical dance. It is a dance-drama performance art
with origins in the Krishna-centered Vaishnavism monasteries of Assam, and attributed to the 15th
century Bhakti movement scholar and saint named Srimanta Sankardev.
Total Marks : 200
Online Prelims TEST - 12 (TEXTBOOK)
( InsightsIAS Mock Test Series for UPSC Preliminary Exam 2020 ) Mark Scored : 0

Q Source: Revision

89 Mangrove forests can be found in which of the following river deltas?


1. Ganga-Brahmaputra
2. Mahanadi
3. Godavari
4. Krishna

Select the correct answer using the codes below.


A. 1 and 2 only
B. 2, 3 and 4 only
C. 1 and 4 only
D. 1, 2, 3 and 4

Your Answer :
Correct Answer : D

Answer Justification :

Justification: In India, the mangrove forests spread over 6,740 sq. km which is 7 per cent of the
world’s mangrove forests.

They are highly developed in the Andaman and Nicobar Islands and the Sunderbans - the largest in
the world, located in the Ganges River delta in Bangladesh and West Bengal.

Other areas of significance are the Mahanadi, the Godavari and the Krishna deltas.

The table below shows that mangroves can be found in every coastal state with West Bengal being
the highest.
Total Marks : 200
Online Prelims TEST - 12 (TEXTBOOK)
( InsightsIAS Mock Test Series for UPSC Preliminary Exam 2020 ) Mark Scored : 0

Q Source: Chapter 5: 11th NCERT: India Physical Environment

90 Which of these authorities approves the sale of government stake in the Public Sector Units (PSUs)?

A. Cabinet Committee on Economic Affairs (CCEA)


B. Governor, Reserve Bank of India
C. Finance Secretary, Ministry of Finance
D. Department of Industrial Policy and Promotion (DIPP)

Your Answer :
Correct Answer : A

Answer Justification :

Justification: Recently, the CCEA has approved sale of government’s more than 51% stake along
with management control in HPCL (Hindustan Petroleum) to ONGC. HPCL will continue as PSU
Total Marks : 200
Online Prelims TEST - 12 (TEXTBOOK)
( InsightsIAS Mock Test Series for UPSC Preliminary Exam 2020 ) Mark Scored : 0

after the acquisition.

Learning: A majority stake sale, if offered to a private enterprise, is termed as a strategic sale that
usually involves shedding management control by the government.

On the other hand, in a minority stake, only part of the stake is offered, and the government retains
both the ownership and the management control.

Q Source: Frequently in news

91 Consider the following about BrahMos Missile.


1. It can be fired from land, sea and air.
2. It is a supersonic missile.
3. It uses atmospheric oxygen as a fuel.
4. It was inducted in the Indian armed forces recently only after India became a member of the Missile
Technology Control Regime (MTCR).

Select the correct answer using the codes below.


A. 1, 2 and 3 only
B. 2, 3 and 4 only
C. 1 and 2 only
D. 3 and 4 only

Your Answer :
Correct Answer : C

Answer Justification :

Justification: Statement 1: It operates on fire and forget principal and is capable of being launched
from land, sea, sub-sea and air against sea and land targets.

Statement 2: It is capable of carrying a warhead and has top supersonic speed of Mach 3 (that is,
three times the speed of sound).

Statement 3: It is two-stage missile, the first one being solid and the second one ramjet liquid
propellant.

Statement 4: The BrahMos Missile already been deployed by Indian Army and Indian Navy in anti-
ship and precision strike roles respectively. The air version is at present undergoing testing.

However, Russia was unable to supply crucial updates to the Brahmos to increase it range due to
non-member restrictions for India under the MTCR regime. Now that India has become a MTCR
member, Russia has supplied these crucial inputs to India.

Q Source: Based on past year UPSC papers

92 In medieval India, takkavi loans were given in order to


Total Marks : 200
Online Prelims TEST - 12 (TEXTBOOK)
( InsightsIAS Mock Test Series for UPSC Preliminary Exam 2020 ) Mark Scored : 0

A. Improve agriculture
B. Build religious shrines
C. Construct flood saving embankments in cities
D. Serve sovereign debt

Your Answer :
Correct Answer : A

Answer Justification :

Learning: Takkavi loans were a measure of reprieve taken after few suppressing moves of Tughlaq.

In order to overcome financial difficulties in his empire, Muhammad bin Tughlaq increased
the land revenue on the farmers of Doab (land between Ganges and Yamuna rivers).
It was an excessive and arbitrary step on the farmers. A severe famine was also ravaging that
region at that time. It had resulted in a serious peasant revolts. The revolts were crushed.
However, the Sultan realized later that adequate relief measures and the promotion of
agriculture were the real solution to the problem.
He launched a scheme by which takkavi loans (loans for cultivation) were given to the farmers
to buy seed and to extend agriculture.

Q Source: Revision: Themes in Indian History – III

93 World's first sanctuary for White Tigers is located in which of these states of India?

A. Gujarat
B. Assam
C. Madhya Pradesh
D. Uttar Pradesh

Your Answer :
Correct Answer : C

Answer Justification :

Justification: In India, white tigers are predominantly found in Rewa, Madhya Pradesh.

In 2016, world’s first White Tiger Safari was inaugurated in Satna district of Madhya Pradesh.

Learning: White tigers are found in other major zoos across India as well, for e.g. Nandankanan
Zoological Park, Odisha.

According to the scientists, white tigers lack pheomelanin, which is responsible for the red-yellow
hue in the skin coat.

Q Source: Current Affairs: last 2 years (Revision):


http://timesofindia.indiatimes.com/india/Worlds-first-sanctuary-for-white-tigers-opens-in-MP/articles
how/51675157.cms
Total Marks : 200
Online Prelims TEST - 12 (TEXTBOOK)
( InsightsIAS Mock Test Series for UPSC Preliminary Exam 2020 ) Mark Scored : 0

94 PRAGATI (Pro-Active Governance And Timely Implementation) is a unique integrating and interactive
platform to
1. Address the grievances of citizens
2. Monitor and review important government projects

Which of the above is/are correct?


A. 1 only
B. 2 only
C. Both 1 and 2
D. None

Your Answer :
Correct Answer : C

Answer Justification :

Justification: Key features of the PRAGATI application are as follows:

It is a three-tier system (PMO, Union Government Secretaries, and Chief Secretaries of the
States);
Prime Minister will hold a monthly programme where he will interact with the Government of
India Secretaries, and Chief Secretaries through Video-conferencing enabled by data and geo-
informatics visuals;
Issues to be flagged before the PM are picked up from the available database regarding
Public Grievances, on-going Programmes and pending Projects;
It will also take into consideration various correspondences to PM’s office by the common
people or from high dignitaries of States and/or developers of public projects;

Q Source: PMO Website

95 India Infrastructure Fund (IIF)


1. is a SEBI-registered domestic venture capital fund
2. sponsored entirely by foreign governments
3. invests only in greenfield projects

Select the correct answer using the codes below.


A. 1 only
B. 2 and 3 only
C. 1 and 3 only
D. None of the above

Your Answer :
Correct Answer : A

Answer Justification :

Justification: Statement 1: It is focused on long-term equity investments in a diversified portfolio of


infrastructure projects.
Total Marks : 200
Online Prelims TEST - 12 (TEXTBOOK)
( InsightsIAS Mock Test Series for UPSC Preliminary Exam 2020 ) Mark Scored : 0

Statement 2: IIF has been sponsored by IDFC Limited (IDFC), along with Citigroup Inc.(Citi) and
India Infrastructure Finance Company Limited (IIFCL) as founder investors.

The establishment of IIF has received the strong support of the Government of India.

Statement 3: IIF’s portfolio is expected to comprise greenfield, brownfield and operational


assets/projects in core infrastructure sub-sectors including transport etc.

The current size of IIF is INR 38 billion with investor commitments from institutional investors in
India, USA, Canada, Europe, Japan and the Middle-East.

China-led Asian Infrastructure Investment Bank (AIIB) has approved $150 million equity investment
loan to the India Infrastructure Fund. This loan will be the bank’s first equity investment to fund
private projects.

Q Source: Sometimes seen in news

96 Under the constitution of India, Sports is a

A. Union Subject
B. State subject
C. Concurrent List subject
D. Residuary list subject

Your Answer :
Correct Answer : B

Answer Justification :

Learning: Sports is a State subject and, therefore, it is primarily the responsibility of the State
Government to promote and develop Sports in the country.

The Central Government complements/supplements the efforts of the State Governments.

Under the revamped Khelo India proposal, there is an exclusive component called “Promotion of
Rural, Indigenous and Tribal Games” for promotion and development of traditional sports.

Q Source: Basics of Polity

97 Most of the cultivable land in India is owned by

A. Small and marginal farmers


B. Medium and Large farmers
C. Scheduled tribes
D. land share is nearly equally divided among all.

Your Answer :
Correct Answer : B
Total Marks : 200
Online Prelims TEST - 12 (TEXTBOOK)
( InsightsIAS Mock Test Series for UPSC Preliminary Exam 2020 ) Mark Scored : 0

Answer Justification :

Justification: A pie-chart from NCERT:

These are some of the important agricultural statistics:

The average holding in India is tiny and often split into scattered pieces. Five major categories of
land holding are: marginal (below 1 ha), small (1-2 ha), semi-medium (2-4 ha), medium (4-10 ha),
large (10ha and above).

According to Census 2011, 67 per cent of holdings were classified as marginal (less than one
hectare) and 18 per cent were classified as small (one-two hectare).
Nearly 80% of the farmers are small and marginal farmers, yet they hold a tiny fraction of the
total landholding.
Total Marks : 200
Online Prelims TEST - 12 (TEXTBOOK)
( InsightsIAS Mock Test Series for UPSC Preliminary Exam 2020 ) Mark Scored : 0

Q Source: Chapter 1: Graph 1.1: 9th NCERT Economics

98 In India, the Prime Minister enjoys a pre-eminent place in the government. Consider the following
with reference to this.
1. The Council of Ministers comes into existence only after the Prime Minister has taken the oath of
office.
2. The resignation of the Prime Minister automatically brings about the dissolution of the Council of
Ministers.

Which of the above is/are correct?


A. 1 only
B. 2 only
C. Both 1 and 2
D. None

Your Answer :
Correct Answer : C

Answer Justification :

Justification: Statement 1: This is because the Council of Ministers cannot exist without the Prime
Minister. He advises the appointment of all the ministers and heads the cabinet meetings. He can
also advise the President to dismiss ministers from the CoM.

Statement 2: The death or resignation of the Prime Minister automatically brings about the
dissolution of the Council of Ministers but the demise, dismissal or resignation of a minister only
creates a ministerial vacancy. This is because the entire CoM is de facto appointed by the PM, and
after his resignation a new CoM is constituted.

Q Source: 11th NCERT: Constitution at work

99 With reference to the World Heritage Cities Programme and associated initiatives, consider the
following:
1. It is a thematic programmes formally approved and monitored by the World Heritage Committee.
2. The programme has provisions for technical assistance to States Parties for the implementation of
new approaches and schemes for urban heritage conservation.
3. IUCN has set up the ‘Historic Urban Landscape initiative’ which is an international working group
comprising, inter alia, UNESCO as an advisory body to the World Heritage Convention.
4. The Organization of World Heritage Cities (OWHC) is an inter-governmental organization of the
municipal corporations of cities in which sites of the UNESCO World Heritage list are located.

Select the correct answer using the codes below.


A. 1 and 2 only
B. 3 and 4 only
C. 1, 2 and 3 only
D. 1, 2, 3 and 4
Total Marks : 200
Online Prelims TEST - 12 (TEXTBOOK)
( InsightsIAS Mock Test Series for UPSC Preliminary Exam 2020 ) Mark Scored : 0

Your Answer :
Correct Answer : A

Answer Justification :

Justification: Statement 1 and 2: It aims to assist States Parties in the challenges of protecting and
managing their urban heritage. The programme is structured along a two-way process, with 1) the
development of a theoretical framework for urban heritage conservation, and 2) the provision of
technical assistance to States Parties for the implementation of new approaches and schemes.

Statement 3: As part of this policy process of conservation, UNESCO (not IUCN) set up the Historic
Urban Landscape initiative, an international working group comprising ICOMOS, IUCN and
ICCROM (as Advisory Bodies to the 1972 World Heritage Convention) and other partner
organizations, including UN-Habitat etc.

Statement 4: OWHC is an international non-profit, non-governmental organization of 250 cities in


which sites of the UNESCO World Heritage list are located.

Learning: It is first Indian city get the honour. The Walled City of Ahmedabad is located on the
eastern banks of Sabarmati river.

It was founded by Sultan Ahmed Shah I of Gujarat Sultanate in in 1411.


It remained the capital of the Gujarat Sultanate and later important political and commercial
centre of Gujarat.
It presents a rich architectural heritage from the sultanate period, notably the Bhadra citadel,
walls and gates of the Fort city and numerous mosques and tombs, as well as important Hindu
and Jain temples of later periods.

Q Source: Based on previous test’s syllabus

100 The organization United Nations (UN) Women works under the overall supervision and aegis of

A. UN General Assembly
B. UN Economic and Social Council
C. Commission on Status of Women
D. All of above

Your Answer :
Correct Answer : D

Answer Justification :

Learning: In 2010, the United Nations General Assembly created UN Women, the United Nations
Entity for Gender Equality and the Empowerment of Women. In doing so, UN Member States took
an historic step in accelerating the Organization’s goals on gender equality and the empowerment
of women.

According to UN General Assembly resolution, which established UN Women, the organization is


Total Marks : 200
Online Prelims TEST - 12 (TEXTBOOK)
( InsightsIAS Mock Test Series for UPSC Preliminary Exam 2020 ) Mark Scored : 0

governed by a multi-tiered intergovernmental governance structure as follows:

“(a) … the General Assembly, the Economic and Social Council and the Commission on the Status of
Women shall constitute the multi-tiered intergovernmental governance structure for the normative
support functions and shall provide normative policy guidance to the Entity;

Q Source:
http://www.un.org/en/sections/about-un/funds-programmes-specialized-agencies-and-others/index.ht
ml

A initiative to provide watermarks and bookmarks free pdfs to you.


Share and Subscribe our telegram channel
@visionpt3652019

https://t.me/visionpt3652019

You might also like